Первый закон ньютона формулировка и формула: Что такое первый закон Ньютона? (статья)

Содержание

Законы Ньютона — простыми словами. Объяснение с примерами

С помощью законов Ньютона можно описать движение любой механической системы, будь то старушка, переходящая дорогу, или робот-пылесос. А может быть, это девятиклассница Соня, которая едет в поезде метро на занятия. При торможении электропоезда на станции Соня некоторое время продолжает по инерции двигаться вперед.

Инерция — явление сохранения постоянной скорости тела при отсутствии действия на него других тел.

С инерцией мы сталкиваемся каждый день:

  • велосипед движется, если перестать крутить педали;
  • бегун не может остановиться сразу после финиша, а пробегает некоторое расстояние;
  • чай в кружке продолжает вращаться, если перестать его размешивать и убрать ложку;
  • дверь способна сама захлопнуться после толчка.

Объяснить это явление можно с помощью первого закона Ньютона, который также называют законом инерции.

Первый закон Ньютона: формулировка

Существуют системы отсчета, называемые инерциальными (ИСО), в которых тело находится в состоянии покоя (V = 0) или движется равномерно и прямолинейно (V = const), если на тело не действуют силы (F = 0) или действие этих сил скомпенсировано (F = 0).

Инерциальные системы отсчета окружают нас повсюду. Например, равномерно спускающийся лифт или тот самый поезд метро, в котором Соня равномерно движется между станциями.

Инерциальные системы отсчета обладают следующими свойствами:

  • тела в таких системах движутся равномерно или находятся в состоянии покоя;
  • при одинаковых начальных условиях тела движутся одинаково;
  • изменение скорости тела происходит в результате действия на него других тел.

Остановимся на последнем свойстве подробнее и рассмотрим пример.

Кот Василий неподвижно спит на батарее. На него определенно действуют силы: со стороны Земли — сила тяжести, направленная вниз, а со стороны батареи — сила реакции опоры, направленная вертикально вверх. Однако изменения скорости Василия не происходит потому, что действие вышеупомянутых сил скомпенсировано.

1-й закон Ньютона не имеет формулы, однако математически его можно описать следующим образом:

,
где — скорость тела [м/с],
— равнодействующая сила [Н].

Равнодействующая сила

— векторная сумма всех сил, действующих на тело. При равномерном прямолинейном движении или в состоянии покоя равнодействующая сила равна нулю.

Вернемся к примеру с котом Василием. До тех пор, пока кота никто не трогает, он находится в состоянии покоя. Когда Соня толкнет Василия с некоторой силой, его скорость изменится. Причем чем большую силу приложит Соня, тем большее ускорение приобретет Василий. Связь между ускорением тела и приложенной силой устанавливает 2-й закон Ньютона.

Практикующий детский психолог Екатерина Мурашова

Бесплатный курс для современных мам и пап от Екатерины Мурашовой. Запишитесь и участвуйте в розыгрыше 8 уроков

Второй закон Ньютона: формулировка

В ИСО ускорение, с которым движется тело, прямо пропорционально равнодействующей всех сил и обратно пропорционально массе этого тела.

Вспомним Соню в поезде метро. Рассмотрим участок разгона электропоезда под действием равнодействующей силы. Согласно 2-му закону Ньютона, чем больше равнодействующая сила, тем большее ускорение приобретет поезд. Под действием той же силы более легкий поезд будет двигаться с бóльшим ускорением.

Второй закон Ньютона: формула

,
где — ускорение [м/с2],
— равнодействующая сила [Н],
— масса [кг].

Важно

Сила и ускорение — величины векторные, их направления всегда совпадают.

Рассмотрим примеры решения задач с использованием второго закона Ньютона.

Задача 1

Уставший Аркаша пришел домой после школы и с силой 4,5 Н горизонтально бросил в сторону кровати рюкзак массой 6 кг. Какое ускорение приобрел рюкзак? Силой сопротивления воздуха можно пренебречь.

Решение.

Сила воздействия Аркаши на рюкзак при горизонтальном броске равна равнодействующей силе. Подставим в формулу 2-го закона Ньютона числа:

м/с2.

Ответ: рюкзак приобрел ускорение 0,75 м/с2.

Задача 2

На рисунке отмечены все силы, действующие на тело. Чему равна равнодействующая сила, если одной клетке соответствует 1 Н?

Решение.

Для определения равнодействующей силы необходимо найти векторную сумму F1, F2 и F3 с помощью правил сложения векторов. Согласно правилу треугольника, чтобы сложить два вектора, нужно последовательно отложить их друг от друга (т. е. начало второго вектора должно совпадать с концом первого).

Сложим силы F2 и F3, лежащие в горизонтальной плоскости. Их сумма имеет длину 3 клетки и направлена вправо в сторону большей силы.

Затем полученную сумму сложим с силой F1 по правилу параллелограмма. Отложим силы F1 и F23 от одной точки, достроим до параллелограмма. Диагональ параллелограмма является искомой суммой ΣF.

По теореме Пифагора найдем гипотенузу:

И вычислим:

Ответ: равнодействующая сила равна 34.

Хотите найти универсальный способ решения всех задач по динамике, успешно справляться с заданиями ОГЭ и даже освоить самую сложную задачу № 30 из ЕГЭ? Тогда записывайте алгоритм. Вот 7 шагов к успеху!

Алгоритм решения задач с использованием 2-го закона Ньютона

  1. Выбрать ИСО.

  2. Отметить на рисунке все силы, действующие на тело.

  3. Записать 2-й закон Ньютона в векторном виде.

  4. Найти проекции сил на координатные оси.

  5. Записать 2-й закон Ньютона в проекциях на координатные оси.

  6. Составить и решить систему уравнений.

  7. Выполнить расчет и записать ответ.

Попробуем применить алгоритм прямо сейчас, чтобы лучше разобраться в каждом шаге.

Задача 3

Серёжа с силой F = 12 Н, приложенной под углом 30°, тянет машинку массой 600 г по шероховатому ламинату, как показано на рисунке. С каким ускорением движется машинка? Коэффициент трения равен 0,1.

Решение.

При решении задачи будем считать машинку материальной точкой. Выберем направления осей, как показано на рисунке, и отметим все действующие в системе силы. На машинку действуют сила тяги Серёжи F, сила тяжести mg, сила трения Fтр, сила реакции опоры N.

Запишем 2-й закон Ньютона в векторном виде:

Определим проекции силы на координатные оси и запишем 2-й закон Ньютона в проекциях на эти оси.

Ox: ma = Fcosα − Fтр; (1)
Oy: 0 = N + Fsinα − mg. (2)
Запишем формулу для силы трения скольжения: Fтр = μN. (3)

Решим полученную систему из трех уравнений. Для этого подставим выражение для силы трения (3) в уравнение (1) и получим:
ma = Fcosα − μN; (1)
0 = N + Fsinα − mg. (2)

Затем выразим в уравнении (2) силу реакции опоры N = mg − Fsinα и подставим полученное выражение в уравнение (1):
ma = Fcosα − μ(mg − Fsinα).

Выразим искомое ускорение:

И вычислим:
м/с2.

Ответ: машинка движется с ускорением 17,3 м/с2.

Как мы уже заметили, тела постоянно взаимодействуют друг с другом. Именно об этом говорит 3-й закон Ньютона.

Третий закон Ньютона: формулировка

Тела действуют друг на друга с силами, направленными вдоль одной прямой, противоположными по направлению и разными по модулю.

Суть закона в том, что сила действия равна силе противодействия. Причем силы имеют одну природу, а приложены они к разным телам.

Действие и противодействие встречаются повсюду:

  • мы притягиваем к себе Землю с той же силой, с какой она притягивает нас;

  • боксер носит перчатки потому, что груша ударяет его с той же силой, что и он;

  • ноутбук давит на стол с той же силой, что и стол на ноутбук;

  • прыжок гребца из лодки непременно вызовет движение лодки в противоположную сторону;

  • лебедь плавает по озеру за счет взаимодействия с водой.

Третий закон Ньютона: формула

,
где — сила, с которой тело 1 действует на тело 2 [Н],
— сила, с которой тело 2 действует на тело 1 [Н].

Для решения задач часто используют комбинацию 2-го и 3-го законов Ньютона, которая имеет следующий вид:

,
где — масса тела 1 [кг],
— масса тела 2 [кг],
— ускорение тела 1 [м/с2],
— ускорение тела 2 [м/с2].

Задача 4

Во время веселых стартов две команды перетягивают канат. Команда «Чемпионы» тянет с максимальной силой 240 Н, а команда «Апельсинки» — с силой 280 Н. С какой силой команды могут натянуть канат, стоя неподвижно на одном месте?

Решение.

Поскольку сила действия равна силе противодействия, а «Чемпионы» могут действовать с силой не более 240 Н, то именно с такой силой команды могут натягивать канат, удерживая его неподвижно.

Ответ: команды могут натягивать канат, стоя неподвижно, с силой 240 Н.

Вот мы и рассмотрели три закона Ньютона. С их помощью любая задача по динамике теперь вам по плечу!

Но это еще не все. Мы подготовили подарок — готовые схемы и формулы по наиболее часто встречающимся типам задач на законы Ньютона.

Онлайн-курсы физики в Skysmart не менее увлекательны, чем наши статьи. На уроках вы изучите немало любопытных физических явлений и научитесь решать самые разнообразные задачи. Ждем вас!

Законы Ньютона – Справочник – 2022

Ньютона физика

Три закона Ньютона лежат в основе классической физики, хотя за прошедшие годы стало понятно, что они — лишь частный случай теории относительности. В нашей статье разбираем формулы и определения законов Ньютона простыми словами

Вплоть до XVII века мировая наука жила в условиях почти религиозной веры в постулаты, заданные великим философом Аристотелем. Покушение на них воспринималось как ересь и безжалостно наказывалось. Доходило даже до инквизиции. В этих условиях деятельность Галилея, Декарта, Ньютона была не только научным, но и человеческим подвигом. Их открытия сегодня могут быть даже переформулированы, не теряя своего смысла и значения.

Про то, как Ньютон открыл закон всемирного тяготения, знают практически все.

Это та самая история про яблоко, которое упало ему на голову. На самом деле, яблоко на голову Ньютона не падало, но все это происходило в осеннем яблоневом саду, где яблоки действительно падали.

А вот как были сформулированы три знаменитых закона Ньютона, ставшие фундаментом классической механики, знают далеко не все. Впервые формулировки этих законов появились в книге Ньютона «Математические начала натуральной философии» (1687 год). Название этого труда достаточно известно именно потому, что в них впервые Ньютон дал определения всех трех законов.

Но перед тем, как формулировки этих законов были напечатаны, много чего произошло. Начиная с Древней Греции, многие мыслители пытались облечь в слова фундаментальные законы движения. Потребовалось несколько веков, чтобы сложились предпосылки для этого. Ближе всего к этому подошел Галилей. Но и ему помешали господствующие в научном сообществе иллюзии. Все были безоговорочно уверены, что небесные тела движутся строго по круговым орбитам, потому что это творение Бога, и это творение должно быть совершенно и безупречно.

Пошатнуть эти иллюзии удалось Кеплеру. Но и он в своих размышлениях пошел не туда.

Гениальность Ньютона заключается в том, что, изучая труды своих великих предшественников, он смог разглядеть неочевидные вещи, которые даже нам кажутся парадоксальными. Именно Ньютон выдвинул революционную идею, что если на тело не действуют никакие силы, то тело может двигаться прямолинейно и равномерно. В условиях Земли это невозможно, так как действует сила земного тяготения. А вот вне Земли — это обычное дело.

Долгие годы размышлений, черновых набросков, сомнений, которые он выражал в письмах своим коллегам, завершились блестящими формулировками всех трех законов. И эти законы по праву носят имя Ньютона. О каждом из этих законов можно написать отдельную статью — настолько велико и многогранно их значение.

Первый закон Ньютона еще называют закон инерции. Фактически он был открыт Галилеем, но именно Исаак Ньютон дал точную его формулировку и включил в число основных законов механики.

Существуют такие системы отсчёта, называемые инерциальными, относительно которых материальная точка при отсутствии внешних воздействий сохраняет величину и направление своей скорости неограниченно долго. Формулы первый закон Ньютона не имеет.

Действие второго закона Ньютона мы можем часто наблюдать в жизни. Возьмём теннисную ракетку и мяч. Если ударить ракеткой по мячу, то мяч приобретёт ускорение равное отношению равнодействующей всех сил к массе.

В инерциальных системах отсчёта ускорение, приобретаемое материальной точкой, прямо пропорционально вызывающей его силе, совпадает с ней по направлению и обратно пропорционально массе материальной точки.

Третий закон Ньютона объясняет, как, например, двигаются утки. Они находятся во взаимодействии с водой, отталкивая ее назад лапками, а сами благодаря ответному действию двигаются вперед.

Материальные точки взаимодействуют друг с другом силами, имеющими одинаковую природу, направленными вдоль прямой, соединяющей эти точки, равными по модулю и противоположными по направлению.

— В школе изучают четыре закона Ньютона. Первые три являются фундаментом классической динамики. С их помощью можно описать любое движение тела, которое движется со скоростью, значительно меньшей скорости света.

Особняком стоит закон всемирного тяготения. Он лежит в основе классической теории гравитации. Этот закон перекликается со вторым законом, касающемся соотношения между ускорением тела, его массой и действующей на него силой. Но все же это разные законы. Так как второй закон Ньютона более универсален, чем закон всемирного тяготения.

— Эти законы настолько фундаментальны, что увидеть их можно практически всюду. Пуля из ружья летит туда, куда толкают ее пороховые газы. Ракета летит туда, куда толкают ее продукты горения ракетного топлива. Мяч летит туда, куда пинает его футболист. Это примеры действия первого закона Ньютона.

Второй закон связан с ускорением, которое приобретает тело под действием силы, действующей на это тело. Например, болид «Формулы 1» разгоняется намного быстрее, чем простой серийный легковой автомобиль. На них действуют разные по величине силы тяги их двигателей. Мощность двигателя в спортивной машине на порядок выше, чем у обыкновенной.

Третий закон Ньютона устанавливает закон взаимодействия тел. Сила действия равна силе противодействия. Например, если чашка кофе стоит на столе и не проваливается сквозь него на пол, значит, стол оказывает достаточное противодействие силе тяжести, действующей на чашку. В результате эти две силы уравновешивают друг друга, и чашка стоит на столе без какого-либо движения.

Утка находится во взаимодействии с водой, отталкивая ее назад лапками, а сама благодаря ответному действию движется вперед. Фото pixabay. com

На самом деле, яблоко на голову Ньютона не падало, но все это происходило в осеннем яблоневом саду, где яблоки действительно падали.

Www. kp. ru

19.09.2017 16:07:38

2017-09-19 16:07:38

Источники:

Https://www. kp. ru/putevoditel/obrazovanie/fizika/zakony-nyutona/

Законы Ньютона (основная школа) » /> » /> . keyword { color: red; }

Ньютона физика

Первый закон Ньютона. Если на тело не действуют силы или их действие скомпенсировано, то данное тело находится в состоянии покоя или равномерного прямолинейного движения.

Свойство тел сохранять свою скорость при отсутствии действия на него других тел называется инерцией. Масса тела – количественная мера его инертности. В СИ она измеряется в килограммах.

Системы отсчета, в которых выполняется первый закон Ньютона, называются инерциальными. Системы отсчета, движущиеся относительно инерциальных с ускорением, называются неинерциальными.

Сила – количественная мера взаимодействия тел. Сила – векторная величина и измеряется в ньютонах (Н). Сила, которая производит на тело такое же действие, как несколько одновременно действующих сил, называется равнодействующей этих сил.

Второй закон Ньютона. Ускорение тела прямо пропорционально равнодействующей сил, приложенных к телу, и обратно пропорционально его массе:

Если два тела взаимодействуют друг с другом, то ускорения этих тел обратно пропорциональны их массам.

Третий закон Ньютона. Силы, с которыми тела взаимодействуют друг с другом, равны по модулю и направлены вдоль одной прямой в противоположные стороны.

В СИ она измеряется в килограммах.

School-collection. edu. ru

28.06.2018 3:04:51

2018-06-28 03:04:51

Источники:

Http://school-collection. edu. ru/catalog/res/14f807b7-509c-b16a-9d9c-de7994a5789d/view/

Законы Ньютона — простыми словами. Объяснение с примерами » /> » /> .keyword { color: red; }

Ньютона физика

Почему при резком торможении автобуса пассажиры движутся вперед? С какой силой действует груша на боксера при ударе? На эти и другие вопросы отвечают законы Ньютона, которые лежат в основе классической механики.

С помощью законов Ньютона можно описать движение любой механической системы, будь то старушка, переходящая дорогу, или робот-пылесос. А может быть, это девятиклассница Соня, которая едет в поезде метро на занятия. При торможении электропоезда на станции Соня некоторое время продолжает по инерции двигаться вперед.

Инерция — явление сохранения постоянной скорости тела при отсутствии действия на него других тел.

С инерцией мы сталкиваемся каждый день:

    велосипед движется, если перестать крутить педали; бегун не может остановиться сразу после финиша, а пробегает некоторое расстояние; чай в кружке продолжает вращаться, если перестать его размешивать и убрать ложку; дверь способна сама захлопнуться после толчка.

Объяснить это явление можно с помощью первого закона Ньютона, который также называют законом инерции.

Первый закон Ньютона: формулировка

Существуют системы отсчета, называемые инерциальными (ИСО), в которых тело находится в состоянии покоя (V = 0) или движется равномерно и прямолинейно (V = const), если на тело не действуют силы (F = 0) или действие этих сил скомпенсировано (F = 0).

Инерциальные системы отсчета окружают нас повсюду. Например, равномерно спускающийся лифт или тот самый поезд метро, в котором Соня равномерно движется между станциями.

Инерциальные системы отсчета обладают следующими свойствами:

    тела в таких системах движутся равномерно или находятся в состоянии покоя; при одинаковых начальных условиях тела движутся одинаково; изменение скорости тела происходит в результате действия на него других тел.

Остановимся на последнем свойстве подробнее и рассмотрим пример.

Кот Василий неподвижно спит на батарее. На него определенно действуют силы: со стороны Земли — сила тяжести, направленная вниз, а со стороны батареи — сила реакции опоры, направленная вертикально вверх. Однако изменения скорости Василия не происходит потому, что действие вышеупомянутых сил скомпенсировано.

1-й закон Ньютона не имеет формулы, однако математически его можно описать следующим образом:

,
Где — скорость тела [м/с],
— равнодействующая сила [Н].

Равнодействующая сила — векторная сумма всех сил, действующих на тело. При равномерном прямолинейном движении или в состоянии покоя равнодействующая сила равна нулю.

Вернемся к примеру с котом Василием. До тех пор, пока кота никто не трогает, он находится в состоянии покоя. Когда Соня толкнет Василия с некоторой силой, его скорость изменится. Причем чем большую силу приложит Соня, тем большее ускорение приобретет Василий. Связь между ускорением тела и приложенной силой устанавливает 2-й закон Ньютона.

Второй закон Ньютона: формулировка

В ИСО ускорение, с которым движется тело, прямо пропорционально равнодействующей всех сил и обратно пропорционально массе этого тела.

Вспомним Соню в поезде метро. Рассмотрим участок разгона электропоезда под действием равнодействующей силы. Согласно 2-му закону Ньютона, чем больше равнодействующая сила, тем большее ускорение приобретет поезд. Под действием той же силы более легкий поезд будет двигаться с бóльшим ускорением.

Второй закон Ньютона: формула

,
Где — ускорение [м/с 2 ],
— равнодействующая сила [Н],
— масса [кг].

Рассмотрим примеры решения задач с использованием второго закона Ньютона.

Задача 1

Уставший Аркаша пришел домой после школы и с силой 4,5 Н горизонтально бросил в сторону кровати рюкзак массой 6 кг. Какое ускорение приобрел рюкзак? Силой сопротивления воздуха можно пренебречь.

Решение.

Сила воздействия Аркаши на рюкзак при горизонтальном броске равна равнодействующей силе. Подставим в формулу 2-го закона Ньютона числа:

Ответ: рюкзак приобрел ускорение 0,75 м/с 2 .

Задача 2

На рисунке отмечены все силы, действующие на тело. Чему равна равнодействующая сила, если одной клетке соответствует 1 Н?

Решение.

Для определения равнодействующей силы необходимо найти векторную сумму F1, F2 и F3 с помощью правил сложения векторов. Согласно правилу треугольника, чтобы сложить два вектора, нужно последовательно отложить их друг от друга (т. е. начало второго вектора должно совпадать с концом первого).

Сложим силы F2 и F3, лежащие в горизонтальной плоскости. Их сумма имеет длину 3 клетки и направлена вправо в сторону большей силы.

Затем полученную сумму сложим с силой F1 по правилу параллелограмма. Отложим силы F1 и F23 от одной точки, достроим до параллелограмма. Диагональ параллелограмма является искомой суммой ΣF.

По теореме Пифагора найдем гипотенузу:

Ответ: равнодействующая сила равна 34.

Хотите найти универсальный способ решения всех задач по динамике, успешно справляться с заданиями ОГЭ и даже освоить самую сложную задачу № 30 из ЕГЭ? Тогда записывайте алгоритм. Вот 7 шагов к успеху!

Алгоритм решения задач с использованием 2-го закона Ньютона

Отметить на рисунке все силы, действующие на тело.

Записать 2-й закон Ньютона в векторном виде.

Найти проекции сил на координатные оси.

Записать 2-й закон Ньютона в проекциях на координатные оси.

Составить и решить систему уравнений.

Выполнить расчет и записать ответ.

Попробуем применить алгоритм прямо сейчас, чтобы лучше разобраться в каждом шаге.

Задача 3

Серёжа с силой F = 12 Н, приложенной под углом 30°, тянет машинку массой 600 г по шероховатому ламинату, как показано на рисунке. С каким ускорением движется машинка? Коэффициент трения равен 0,1.

Решение.

При решении задачи будем считать машинку материальной точкой. Выберем направления осей, как показано на рисунке, и отметим все действующие в системе силы. На машинку действуют сила тяги Серёжи F, сила тяжести mg, сила трения Fтр, сила реакции опоры N.

Запишем 2-й закон Ньютона в векторном виде:

Определим проекции силы на координатные оси и запишем 2-й закон Ньютона в проекциях на эти оси.

Ox: ma = Fcosα − Fтр; (1)
Oy: 0 = N + Fsinα − mg. (2)
Запишем формулу для силы трения скольжения: Fтр = μN. (3)

Решим полученную систему из трех уравнений. Для этого подставим выражение для силы трения (3) в уравнение (1) и получим:
Ma = Fcosα − μN; (1)
0 = N + Fsinα − mg. (2)

Затем выразим в уравнении (2) силу реакции опоры N = mg − Fsinα и подставим полученное выражение в уравнение (1):
Ma = Fcosα − μ(mg − Fsinα).

Выразим искомое ускорение:

И вычислим:
М/с 2 .

Ответ: машинка движется с ускорением 17,3 м/с 2 .

Как мы уже заметили, тела постоянно взаимодействуют друг с другом. Именно об этом говорит 3-й закон Ньютона.

Третий закон Ньютона: формулировка

Тела действуют друг на друга с силами, направленными вдоль одной прямой, противоположными по направлению и разными по модулю.

Суть закона в том, что сила действия равна силе противодействия. Причем силы имеют одну природу, а приложены они к разным телам.

Действие и противодействие встречаются повсюду:

Мы притягиваем к себе Землю с той же силой, с какой она притягивает нас;

Боксер носит перчатки потому, что груша ударяет его с той же силой, что и он;

Ноутбук давит на стол с той же силой, что и стол на ноутбук;

Прыжок гребца из лодки непременно вызовет движение лодки в противоположную сторону;

Лебедь плавает по озеру за счет взаимодействия с водой.

Третий закон Ньютона: формула

,
Где — сила, с которой тело 1 действует на тело 2 [Н],
— сила, с которой тело 2 действует на тело 1 [Н].

Для решения задач часто используют комбинацию 2-го и 3-го законов Ньютона, которая имеет следующий вид:

,
Где — масса тела 1 [кг],
— масса тела 2 [кг],
— ускорение тела 1 [м/с 2 ],
— ускорение тела 2 [м/с 2 ].

Задача 4

Во время веселых стартов две команды перетягивают канат. Команда «Чемпионы» тянет с максимальной силой 240 Н, а команда «Апельсинки» — с силой 280 Н. С какой силой команды могут натянуть канат, стоя неподвижно на одном месте?

Решение.

Поскольку сила действия равна силе противодействия, а «Чемпионы» могут действовать с силой не более 240 Н, то именно с такой силой команды могут натягивать канат, удерживая его неподвижно.

Ответ: команды могут натягивать канат, стоя неподвижно, с силой 240 Н.

Вот мы и рассмотрели три закона Ньютона. С их помощью любая задача по динамике теперь вам по плечу!

Но это еще не все. Мы подготовили подарок — готовые схемы и формулы по наиболее часто встречающимся типам задач на законы Ньютона.

Онлайн-курсы физики в Skysmart не менее увлекательны, чем наши статьи. На уроках вы изучите немало любопытных физических явлений и научитесь решать самые разнообразные задачи. Ждем вас!

И вычислим м с 2.

Skysmart. ru

12.01.2020 0:59:16

2020-01-12 00:59:16

Источники:

Https://skysmart. ru/articles/physics/zakony-nyutona

Динамика поступательного движения материальной точки. Законы Ньютона

Динамика материальной точки и поступательного движения твердого тела

Первый закон Ньютона. Масса. Сила

Первый закон Ньютона : всякая материальная точка (тело) сохраняет состояние покоя или равномерного прямолинейного движения до тех пор, пока воздействие со стороны других тел не заставит ее изменить это состояние . Стремление тела сохранять состояние покоя или равномерного прямолинейного движения называется инертностью . Поэтому первый закон Ньютона называют также законом инерции .

Первый закон Ньютона выполняется не во всякой системе отсчета, а те системы, по отношению к которым он выполняется, называются инерциальными системами отсчета .

Масса тела – физическая величина, являющаяся одной из основных характеристик материи, определяющая ее инерционные (инертная масса ) и гравитационные (гравитационная масса ) свойства. В настоящее время можно считать доказанным, что инертная и гравитационная массы равны друг другу (с точностью, не меньшей 10 –12 их значения).

Итак, сила – это векторная величина, являющаяся мерой механического воздействия на тело со стороны других тел или полей, в результате которого тело приобретает ускорение или изменяет свою форму и размеры.

Второй закон Ньютона

Второй закон Ньютона – основной закон динамики поступательного движения – от­вечает на вопрос, как изменяется механическое движение материальной точки (тела) под действием приложенных к ней сил.

а ~ F = const ) . (6.1)

а ~ 1 /т (F = const) . (6.2)

а = kF / m . (6.3)

В СИ коэффициент пропорциональности k = 1. Тогда

(6.4)

(6.5)

Векторная величина

(6.6)

численно равная произведению массы материальной точки на ее скорость и имеющая направление скорости, называется импульсом (количеством движения) этой материаль­ной точки.

Подставляя (6.6) в (6.5), получим

(6.7)

Выражение (6.7) называется уравнением движения материальной точки .

Единица силы в СИ – ньютон (Н): 1 Н – сила, которая массе 1 кг сообщает ускорение 1 м/с 2 в направлении действия силы:

1 Н = 1 кг м/с 2 .

Второй закон Ньютона справедлив только в инерциальных системах отсчета. Первый закон Ньютона можно получить из второго.

В механике большое значение имеет принцип независимости действия сил : если на материальную точку действует одновременно несколько сил, то каждая из этих сил сообщает материальной точке ускорение согласно второму закону Ньютона, как будто других сил не было.

Третий закон Ньютона

Взаимодействие между материальными точками (телами) определяется третьим зако­ном Ньютона .

F 12 = – F 21 , (7.1)

Третий закон Ньютона позволяет осуществить переход от динамики отдельной материальной точки к динамике системы материальных точек.

Силы трения

В меха­нике мы будем рассматривать различные силы: трения, упругости, тяготения.

Силы трения , которые препятствуют скольжению соприкасающихся тел друг относительно друга.

Внешним трением называется трение, возникающее в плоскости касания двух соприкасающихся тел при их относительном перемещении.

В зависимости от характера их относительного движения говорят о трении скольжения , качения или верчения .

Внутренним трением называется трение между частями одного и того же тела, например между различными слоями жидкости или газа. Если тела скользят относительно друг друга и разделены прослойкой вязкой жидкости (смазки), то трение происходит в слое смазки. В таком случае говорят о гидродинамическом трении (слой смазки достаточно толстый) и граничном трении (толщина смазоч­ной прослойки 0,1 мкм и меньше).

Сила трения скольжения F тр пропорциональна силе N нормального давления, с которой одно тело действует на другое:

F тр = f N ,

где f – коэффициент трения скольжения, зависящий от свойств соприкасающихся поверхностей.

В пре­дельном случае (начало скольжения тела) F =F тр. или P sin  0 = f N = f P cos  0 , откуда

f = tg 0 .

Для гладких поверхностей определенную роль начинает играть межмолекулярное притяжение. Для них применяется закон трения скольжения

F тр = f ист (N + Sp 0 ) ,

где р 0 добавочное давление, обусловленное силами межмолекулярного притяжения, которые быстро уменьшаются с увеличением расстояния между частицами; S пло­щадь контакта между телами; f ист – истинный коэффициент трения скольжения.

Радикальным способом уменьшения силы трения является замена трения скольже­ния трением качения (шариковые и роликовые подшипники и т. д.). Сила трения качения определяется по закону, установленному Кулоном:

F тр = f к N / r , (8.1)

где r – радиус катящегося тела; f к – коэффициент трения качения, имеющий размер­ность dim f к =L. Из (8.1) следует, что сила трения качения обратно пропорциональна радиусу катящегося тела.

Закон сохранения импульса. Центр масс

Совокуп­ность материальных точек (тел), рассматриваемых как единое целое, называется механической системой . Силы взаимодействия между материальными точками механичес­кой системы называются – внутренними . Силы, с которыми на материальные точки системы действуют внешние тела, называются внешними . Механическая система тел, на которую не действуют внешние силы, называется замкнутой (или изолированной ). Если мы имеем механическую систему, состоящую из многих тел, то, согласно третьему закону Ньютона, силы, действующие между этими телами, будут равны и проти­воположно направлены, т. е. геометрическая сумма внутренних сил равна нулю.

Запишем второй закон Ньютона для каждого из n тел механической системы:

Складывая почленно эти уравнения, получаем

Но так как геометрическая сумма внутренних сил механической системы по третьему закону Ньютона равна нулю, то

(9. 1)

где – импульс системы. Таким образом, производная по времени от им­пульса механической системы равна геометрической сумме внешних сил, действующих на систему.

В случае отсутствия внешних сил (рассматриваем замкнутую систему)

Последнее выражение и является законом сохранения импульса : импульс замкнутой системы сохраняется, т. е. не изменяется с течением времени.

Эксперименты доказывают, что он выпол­няется и для замкнутых систем микрочастиц (они подчиняются законам квантовой механики). Этот закон носит универсальный характер, т. е. закон сохранения импуль­са – фундаментальный закон природы.

Закон сохранения импульса является следствием определенного свойства симмет­рии пространства – его однородности. Однородность пространства заключается в том, что при параллельном переносе в пространстве замкнутой системы тел как целого ее физические свойства и законы движения не изменяются, иными словами, не зависят от выбора положения начала координат инерциальной системы отсчета.

Центром масс (или центром инерции ) системы материальных точек называется воображаемая точка С , положение которой характеризует распределение массы этой системы. Ее ра­диус-вектор равен

где m i и r i – соответственно масса и радиус-вектор i -й материальной точки; n – число материальных точек в системе; – масса системы. Скорость центра масс

Учитывая, что pi = m i v i , a есть импульс р системы, можно написать

(9.2)

т. е. импульс системы равен произведению массы системы на скорость ее центра масс.

Подставив выражение (9.2) в уравнение (9.1), получим

(9.3)

т. е. центр масс системы движется как материальная точка, в которой сосредоточена масса всей системы и на которую действует сила, равная геометрической сумме всех внешних сил, приложенных к системе. Выражение (9.3) представляет собой закон движения центра масс.

1. Инерциальные системы отсчета. Законы Ньютона. Масса, импульс, сила. Уравнение движения материальной точки.

2. Понятие замкнутой системы. Закон сохранения импульса. Центр масс механической системы, закон движения центра масс.

3. Движение тел переменной массы. Уравнение Мещерского. Формула Циолковского.

Цели:

· ввести понятия инерциальной и неинерциальной систем отсчета, массы, импульса, силы, замкнутой системы;

· изучить законы Ньютона;

· вывести и сформулировать закон сохранения импульса;

· описать движение тел переменной массы;

· вывести уравнение Мещерского и формулу Циолковского.

Литература:

1. Трофимова Т.И. Курс физики: учебное пособие для инженерно-технических специальностей вузов – М.: Academia, 2006, 2007 и 2008.

2. Грабовский Р. И. Курс физики [Электронный ресурс]: учебное пособие / Р. И. Грабовский – Санкт-Петербург [и др.]: Лань, 2012.

3. Зисман Г. А. Курс общей физики [Электронный ресурс]: [учебное пособие для студентов высших учебных заведений, обучающихся по техническим, естественнонаучным и педагогическим направлениям и специальностям]: В 3-х т. / Г. А. Зисман, О. М. Тодес – Санкт-Петербург [и др.]: Лань, 2007- Т. 2: Электричество и магнетизм.

4. Ливенцев Н.М. Курс физики [Электронный ресурс]: учебное пособие – СПб: Лань, 2012.

5. Бабаев В.С., Легуша Ф.Ф. Корректирующий курс физики [Электронный ресурс] – СПб: Лань, 2011.

6. Калашников Н. П. Основы физики: учебник для вузов: в 2-х т / Н. П. Калашников, М. А. Смондырев – М.: Дрофа, 2007.

7. Рогачев Н. М. Курс физики [Электронный ресурс]: [учебное пособие для студентов вузов, обучающихся в области техники и технологий] / Н. М. Рогачев – Санкт-Петербург [и др.]: Лань, 2010.

8. Александров И.В. и др. Современная физика [Электронный ресурс]: учебное пособие для студентов всех форм обучения, обучающихся по техническим и технологическим направлениям и специальностям – Уфа: УГАТУ, 2008.

Динамика материальной точки и поступательного движения твердого тела

Первый закон Ньютона. Масса. Сила

Динамика является основным разделом механики, в ее основе лежат три закона Ньютона, сформулированные им в 1687 г. Законы Ньютона играют исключительную роль в механике и являются (как и все физические законы) обобщением результатов огромного человеческого опыта. Их рассматривают как систему взаимосвязанных законов и опытной проверке подвергают не каждый отдельный закон, а всю систему в целом.

Первый закон Ньютона : всякая материальная точка (тело) сохраняет состояние покоя или равномерного прямолинейного движения до тех пор, пока воздействие со стороны других тел не заставит ее изменить это состояние . Стремление тела сохранять состояние покоя или равномерного прямолинейного движения называется инертностью . Поэтому первый закон Ньютона называют также законом инерции .

Механическое движение относительно, и его характер зависит от системы отсчета. Первый закон Ньютона выполняется не во всякой системе отсчета, а те системы, по отношению к которым он выполняется, называются инерциальными системами отсчета . Инерциальной системой отсчета является такая система отсчета, относительно которой материальная точка, свободная от внешних воздействий, либо покоится, либо движется равномерно и прямолинейно. Первый закон Ньютона утверждает существование инерциальных систем отсчета.

Опытным путем установлено, что инерциальной можно считать гелиоцентрическую (звездную) систему отсчета (начало координат находится в центре Солнца, а оси проведаны в направлении определенных звезд). Система отсчета, связанная с Землей, строго говоря, неинерциальна, однако эффекты, обусловленные ее неинерциальностью (Земля вращается вокруг собственной оси и вокруг Солнца), при решении многих задач пренебрежимо малы, и в этих случаях ее можно считать инерциальной.

Из опыта известно, что при одинаковых воздействиях различные тела неодинаково изменяют скорость своего движения, т.е., иными словами, приобретают различные ускорения. Ускорение зависит не только от величины воздействия, но и от свойств самого тела (от его массы).

Масса тела – физическая величина, являющаяся одной из основных характеристик материи, определяющая ее инерционные (инертная масса ) и гравитационные (гравитационная масса ) свойства. В настоящее время можно считать доказанным, что инертная и гравитационная массы равны друг другу (с точностью, не меньшей 10 –12 их значения).

Чтобы описывать воздействия, упоминаемые в первом законе Ньютона, вводят понятие силы. Под действием сил тела либо изменяют скорость движения, т. е. приобретают ускорения (динамическое проявление сил), либо деформируются, т. е. изменяют свою форму и размеры (статическое проявление сил). В каждый момент времени сила характеризуется числовым значением, направлением в пространстве и точкой приложения. Итак, сила – это векторная величина, являющаяся мерой механического воздействия на тело со стороны других тел или полей, в результате которого тело приобретает ускорение или изменяет свою форму и размеры.

Второй закон Ньютона

Второй закон Ньютона – основной закон динамики поступательного движения – от­вечает на вопрос, как изменяется механическое движение материальной точки (тела) под действием приложенных к ней сил.

Если рассмотреть действие различных сил на одно и то же тело, то оказывается, что ускорение, приобретаемое телом, всегда прямо пропорционально равнодействующей приложенных сил:

а ~ F (т = const) . (6.1)

При действии одной и той же силы на тела с разными массами их ускорения оказываются различными, а именно

а ~ 1/т (F = const) . (6.2)

Используя выражения (6.1) и (6.2) и учитывая, что сила и ускорение-величины векторные, можем записать

а = kF/m. (6.3)

Соотношение (6.3) выражает второй закон Ньютона: ускорение, приобретаемое материальной точкой (телом), пропорционально вызывающей его силе, совпадает с нею по направлению и обратно пропорционально массе материальной точки (тела).

В СИ коэффициент пропорциональности k= 1. Тогда

(6.4)

Учитывая, что масса материальной точки (тела) в классической механике есть величина постоянная, в выражении (6.4) ее можно внести под знак производной:

Векторная величина

численно равная произведению массы материальной точки на ее скорость и имеющая направление скорости, называется импульсом (количеством движения) этой материаль­ной точки.

Подставляя (6.6) в (6.5), получим

Это выражение – более общая формулировка второго закона Ньютона : скорость изме­нения импульса материальной точки равна действующей на нее силе. Выражение (6.7) называется уравнением движения материальной точки .

Единица силы в СИ – ньютон (Н): 1 Н – сила, которая массе 1 кг сообщает ускорение 1 м/с 2 в направлении действия силы:

1 Н = 1 кг×м/с 2 .

Второй закон Ньютона справедлив только в инерциальных системах отсчета. Первый закон Ньютона можно получить из второго. Действительно, в случае равенст­ва нулю равнодействующей сил (при отсутствии воздействия на тело со стороны других тел) ускорение (см. (6.3)) также равно нулю. Однако первый закон Ньютона рассматривается как самостоятельный закон (а не как следствие второго закона), так как именно он утверждает существование инерциальных систем отсчета, в которых только и выполняется уравнение (6.7).

В механике большое значение имеет принцип независимости действия сил : если на материальную точку действует одновременно несколько сил, то каждая из этих сил сообщает материальной точке ускорение согласно второму закону Ньютона, как будто других сил не было. Согласно этому принципу, силы и ускорения можно разлагать на составляющие, использование которых приводит к существенному упрощению решения задач. Например, на рис. 10 действующая сила F=m a разложена на два компонен­та: тангенциальную силу F t , (направлена по касательной к траектории) и нормальную силу F n (направлена по нормали к центру кривизны). Используя выражения и , а также , можно записать:

Если на материальную точку действует одновременно несколько сил, то, согласно принципу независимости действия сил, под F во втором законе Ньютона понимают результирующую силу.

Третий закон Ньютона

Взаимодействие между материальными точками (телами) определяется третьим зако­ном Ньютона : всякое действие материальных точек (тел) друг на друга носит характер взаимодействия; силы, с которыми действуют друг на друга материальные точки, всегда равны по модулю, противоположно направлены и действуют вдоль прямой, соединяющей эти точки:

F 12 = – F 21 , (7. 1)

где F 12 – сила, действующая на первую материальную точку со стороны второй;

F 21 – сила, действующая на вторую материальную точку со стороны первой. Эти силы приложены к разным материальным точкам (телам), всегда действуют парами и явля­ются силами одной природы.

Третий закон Ньютона позволяет осуществить переход от динамики отдельной материальной точки к динамике системы материальных точек. Это следует из того, что и для системы материальных точек взаимодействие сводится к силам парного взаимодействия между материальными точками.

Силы трения

Обсуждая до сих пор силы, мы не интересовались их происхождением. Однако в меха­нике мы будем рассматривать различные силы: трения, упругости, тяготения.

Из опыта известно, что всякое тело, движущееся по горизонтальной поверхности другого тела, при отсутствии действия на него других сил с течением времени замедля­ет свое движение и в конце концов останавливается. Это можно объяснить существова­нием силы трения , которая препятствует скольжению соприкасающихся тел друг относительно друга. Силы трения зависят от относительных скоростей тел. Силы трения могут быть разной природы, но в результате их действия механическая энергия всегда превращается во внутреннюю энергию соприкасающихся тел.

Различают внешнее (сухое) и внутреннее (жидкое или вязкое) трение. Внешним трением называется трение, возникающее в плоскости касания двух соприкасающихся тел при их относительном перемещении. Если соприкасающиеся тела неподвижны друг относительно друга, говорят о трении покоя, если же происходит относительное перемещение этих тел, то в зависимости от характера их относительного движения говорят о трении скольжения , качения или верчения .

Внутренним трением называется трение между частями одного и того же тела, например между различными слоями жидкости или газа, скорости которых меняются от слоя к слою. В отличие от внешнего трения здесь отсутствует трение покоя. Если тела скользят относительно друг друга и разделены прослойкой вязкой жидкости (смазки), то трение происходит в слое смазки. В таком случае говорят о гидродинамическом трении (слой смазки достаточно толстый) и граничном трении (толщина смазоч­ной прослойки »0,1 мкм и меньше).

Обсудим некоторые закономерности внешнего трения. Это трение обусловлено шероховатостью соприкасающихся поверхностей; в случае же очень гладких поверх­ностей трение обусловлено силами межмолекулярного притяжения.

Рассмотрим лежащее на плоскости тело (рис. 11), к которому приложена горизон­тальная сила F. Тело придет в движение лишь тогда, когда приложенная сила F будет больше силы трения F тр. Французские физики Г. Амонтон (1663-1705) и Ш. Кулон (1736-1806) опытным путем установили следующий закон : сила трения скольжения F тр пропорциональна силе N нормального давления, с которой одно тело действует на другое:

F тр = f N ,

где f – коэффициент трения скольжения, зависящий от свойств соприкасающихся поверхностей.

Найдем значение коэффициента трения. Если тело находится на наклонной плоско­сти с углом наклона a (рис.12), то оно приходит в движение, только когда тангенциаль­ная составляющая F силы тяжести Р больше силы трения F тр. Следовательно, в пре­дельном случае (начало скольжения тела) F =F тр. или P sin a 0 = f N = f P cos a 0 ,откуда

f = tga 0 .

Таким образом, коэффициент трения равен тангенсу угла a 0 , при котором начинается скольжение тела по наклонной плоскости.

Для гладких поверхностей определенную роль начинает играть межмолекулярное притяжение. Для них применяется закон трения скольжения

F тр = f ист (N + Sp 0) ,

где р 0 добавочное давление, обусловленное силами межмолекулярного притяжения, которые быстро уменьшаются с увеличением расстояния между частицами; S – пло­щадь контакта между телами; f ист – истинный коэффициент трения скольжения.

Трение играет большую роль в природе и технике. Благодаря трению движется транспорт, удерживается забитый в стену гвоздь и т. д.

В некоторых случаях силы трения оказывают вредное действие и поэтому их надо уменьшать. Для этого на трущиеся поверхности наносят смазку (сила трения уменьша­ется примерно в 10 раз), которая заполняет неровности между этими поверхностями и располагается тонким слоем между ними так, что поверхности как бы перестают касаться друг друга, а скользят друг относительно друга отдельные слои жидкости. Таким образом, внешнее трение твердых тел заменяется значительно меньшим внут­ренним трением жидкости.

Радикальным способом уменьшения силы трения является замена трения скольже­ния трением качения (шариковые и роликовые подшипники и т. д.). Сила трения качения определяется по закону, установленному Кулоном:

F тр =f к N/r , (8.1)

где r – радиус катящегося тела; f к – коэффициент трения качения, имеющий размер­ность dim f к =L. Из (8.1) следует, что сила трения качения обратно пропорциональна радиусу катящегося тела.

Закон сохранения импульса. Центр масс

Для вывода закона сохранения импульса рассмотрим некоторые понятия. Совокуп­ность материальных точек (тел), рассматриваемых как единое целое, называется механической системой . Силы взаимодействия между материальными точками механичес­кой системы называются – внутренними . Силы, с которыми на материальные точки системы действуют внешние тела, называются внешними . Механическая система тел, на которую не действуют внешние силы, называется замкнутой (или изолированной ). Если мы имеем механическую систему, состоящую из многих тел, то, согласно третьему закону Ньютона, силы, действующие между этими телами, будут равны и проти­воположно направлены, т. е. геометрическая сумма внутренних сил равна нулю.

Рассмотрим механическую систему, состоящую из n тел, масса и скорость которых соответственно равны m 1 , m 2 , .m n , и v 1 , v 2 ,…, v n . Пусть – равнодейст­вующие внутренних сил, действующих на каждое из этих тел, a – равно­действующие внешних сил. Запишем второй закон Ньютона для каждого из n тел механической системы:

Складывая почленно эти уравнения, получаем

Но так как геометрическая сумма внутренних сил механической системы по третьему закону Ньютона равна нулю, то

(9.1)

где – импульс системы. Таким образом, производная по времени от им­пульса механической системы равна геометрической сумме внешних сил, действующих на систему.

В случае отсутствия внешних сил (рассматриваем замкнутую систему)

Последнее выражение и является законом сохранения импульса : импульс замкнутой системы сохраняется, т. е. не изменяется с течением времени.

Закон сохранения импульса справедлив не только в классической физике, хотя он и получен как следствие законов Ньютона. Эксперименты доказывают, что он выпол­няется и для замкнутых систем микрочастиц (они подчиняются законам квантовой механики). Этот закон носит универсальный характер, т. е. закон сохранения импуль­са – фундаментальный закон природы.

Закон сохранения импульса является следствием определенного свойства симмет­рии пространства – его однородности. Однородность пространства заключается в том, что при параллельном переносе в пространстве замкнутой системы тел как целого ее физические свойства и законы движения не изменяются, иными словами, не зависят от выбора положения начала координат инерциальной системы отсчета.

Отметим, что, согласно (9.1), импульс сохраняется и для незамкнутой системы, если геометрическая сумма всех внешних сил равна нулю.

В механике Галилея-Ньютона из-за независимости массы от скорости импульс системы может быть выражен через скорость ее центра масс. Центром масс (или центром инерции ) системы материальных точек называется воображаемая точка С ,положение которой характеризует распределение массы этой системы. Ее ра­диус-вектор равен

где m i и r i – соответственно масса и радиус-вектор i -й материальной точки; n – число материальных точек в системе; – масса системы. Скорость центра масс

Учитывая, что pi = m i v i , a есть импульс р системы, можно написать

т. е. импульс системы равен произведению массы системы на скорость ее центра масс.

Подставив выражение (9.2) в уравнение (9.1), получим

(9.3)

т. е. центр масс системы движется как материальная точка, в которой сосредоточена масса всей системы и на которую действует сила, равная геометрической сумме всех внешних сил, приложенных к системе. Выражение (9.3) представляет собойзакон движения центра масс.

В соответствии с (9.2) из закона сохранения импульса вытекает, что центр масс замкнутой системы либо движется прямолинейно и равномерно, либо остается непо­движным.

Уравнение движения тела переменной массы

Движение некоторых тел сопровождается изменением их массы, например масса ракеты уменьшается вследствие истечения газов, образующихся при сгорании топлива, и т. п.

Выведем уравнение движения тела переменной массы на примере движения ракеты. Если в момент времени t масса ракеты m , а ее скорость v, то по истечении времени dt ее масса уменьшится на dm и станет равной т – dm, а скорость станет равной v + dv. Изменение импульса системы за отрезок времени dt

где u – скорость истечения газов относительно ракеты. Тогда

(учли, что dm dv – малый высшего порядка малости по сравнению с остальными). Если на систему действуют внешние силы, то dp=Fdt , поэтому

(10.1)

Второе слагаемое в правой части (10.1) называютреактивной силой Fp. Если u про­тивоположен v по направлению, то ракета ускоряется, а если совпадает с v, то тормозится.

Таким образом, мы получилиуравнение движения тела переменной массы

которое впервые было выведено И. В. Мещерским (1859-1935).

Идея применения реактивной силы для создания летательных аппаратов высказы­валась в 1881 г. Н. И. Кибальчичем (1854-1881). К. Э. Циолковский (1857-1935) в 1903 г. опубликовал статью, где предложил теорию движения ракеты и основы теории жидкостного реактивного двигателя. Поэтому его считают основателем отече­ственной космонавтики.

Применим уравнение (10.1) к движению ракеты, на которую не действуют никакие внешние силы. Полагая F=0 и считая, что скорость выбрасываемых газов относитель­но ракеты постоянна (ракета движется прямолинейно), получим

Значение постоянной интегрирования С определим из начальных условий. Если в на­чальный момент времени скорость ракеты равна нулю, а ее стартовая масса m 0 , то С = u ln(m 0). Следовательно,

v = u ln (m 0 /m ). (10.3)

Это соотношение называетсяформулой Циолковского. Она показывает, что: 1) чем больше конечная масса ракеты т, тем больше должна быть стартовая масса ракеты m 0 ; 2) чем больше скорость истечения и газов, тем больше может быть конечная масса при данной стартовой массе ракеты.

Выражения (10.2) и (10.3) получены для нерелятивистских движений, т. е. для случаев, когда скорости v и u малы по сравнению со скоростью с распространения света в вакууме.

Контрольные вопросы

Второй закон Ньютона – основной закон динамики поступательного движения – отвечает на вопрос, как изменяется механическое движение материального объекта (точки, тела) под действием приложенных к нему сил.
В динамике рассматриваются два типа задач, решения которых находятся на основе второго закона Ньютона. Задачи первого типа состоят в том, чтобы, зная движение тела, определить действующие на него силы. Классическим примером решения такой задачи является открытие Ньютоном закона всемирного тяготения: зная установленные Кеплером на основании результатов наблюдений законы движений планет, Ньютон доказал, что это движение происходит под действием силы, обратно пропорциональной квадрату расстояния между планетой и Солнцем.
Задачи второго типа являются в динамике основными и состоят в том, чтобы по действующим на тело силам определить закон его движения (уравнение движения). Для решения этих задач необходимо знать начальные условия, т.е. положение и скорость тела в момент начала его движения под действием заданных сил. Примерами таких задач являются следующие: а) по величине и направлению скорости снаряда в момент его вылета из канала ствола и действующим на снаряд при его движении силе тяжести и силе сопротивления воздуха найти закон движения снаряда, в частности его траекторию, горизонтальную дальность полета, время движения до цели; б) по известным скорости автомобиля в момент начала торможения и силе торможения найти время движения и путь до остановки.
Второй закон Ньютона формулируется следующим образом: ускорение, приобретаемое материальной точкой (телом), прямо пропорционально действующей силе, совпадает с нею по направлению и обратно пропорционально массе материальной точки (тела) :

Где k – коэффициент пропорциональности, зависящий от выбора системы единиц. В международной системе (СИ) k =1, поэтому

(2.4)

Второй закон Ньютона обычно записывается в следующей форме:


или

(2. 5)

Вектор mv=p называется импульсом или количеством движения . В отличие от ускорения и скорости, импульс является характеристикой движущегося тела, отражающей не только кинематическую меру движения (скорость), но и его важнейшее динамическое свойство – массу.

Таким образом, можно записать:

(2.6)

Выражение (2.6) является более общей формулировкой второго закона Ньютона: скорость изменения импульса материальной точки равна действующей на нее силе .
Это уравнение называется уравнением движения материальной точки .
Единица силы в системе СИ – ньютон (Н):
1 Н – это сила, которая телу массой в 1 кг сообщает ускорение 1 м/с 2 в направлении действия силы:

1 Н = 1 кг*1 м/с 2 .
При действии на материальную точку нескольких сил справедлив принцип независимости действия сил : если на материальную точку действуют одновременно несколько сил, то каждая из этих сил сообщает материальной точке ускорение, определяемое вторым законом Ньютона так, как если бы других сил не было:


где сила называется равнодействующей сил или результирующей силой .
Таким образом, если на тело действует одновременно несколько сил, то, согласно принципу независимости действия сил, под силой F во втором законе Ньютона понимают результирующую силу.
Второй закон Ньютона справедлив только в инерциальных системах отсчета. Первый закон Ньютона можно получить из второго закона: в случае равенства нулю равнодействующей силы ускорение также равно нулю, т.е. тело находится в покое или движется равномерно.

Глава 2. ЭЛЕМЕНТЫ ДИНАМИКИ

Динамика изучает движение тел с учетом тех причин (взаимодействий между телами), которые обусловливают тот или иной характер движения. В основе классической (ньютоновской) механики лежат три закона динамики, сформулированные И. Ньютоном в XVII в. Законы Ньютона возникли в результате обобщения большого количества опытных фактов. Правильность их подтверждается совпадением с опытом тех следствий, которые из них вытекают.

Первый закон Ньютона формулируется следующим образом: всякое тело находится в состоянии покоя или равномерного и прямолинœейного движения, пока воздействие со стороны других тел не заставит его изменить это состояние. Оба названных состояния объединяются тем, что ускорение тела равно нулю.

Учитывая, что характер движения зависит от выбора системы отсчета͵ следует сделать вывод, что первый закон Ньютона выполняется не во всякой системе отсчета. Система отсчета͵ в которой выполняется первый закон Ньютона, принято называть инœерциальной. Сам закон называют законом инœерции. Система отсчета͵ в которой первый закон Ньютона не выполняется, принято называть неинœерциальной. Любая система отсчета͵ движущаяся равномерно и прямолинœейно относительно инœерциальной системы, также является системой инœерциальной. По этой причине инœерциальных систем существует бесконечное множество.

Свойство тел сохранять состояние покоя или равномерного и прямолинœейного движения принято называть инœертностью (инœерцией). Мерой инœертности тела является его масса m . Она не зависит от скорости движения тела. За единицу массы принят килограмм (кг) – масса эталонного тела.

В случае если состояние движения тела или его форма и размеры меняются, то говорят, что на тело действуют другие тела. Мерой взаимодействия тел служит сила . Всякая сила проявляется как результат действия одного тела на другое, сводящийся к появлению у тела ускорения или его деформации.

Второй закон Ньютона: результирующая сила, действующая на тело, равна произведению массы этого тела на его ускорение:

Так как масса является скаляром, то из формулы (6.1) следует, что .

На основании этого закона вводится единица силы – ньютон (Н): .

Второй закон Ньютона справедлив только в инœерциальных системах отсчета.

Заменим ускорение в уравнении (6.1) производной скорости по времени:

Векторная величина

принято называть импульсом тела .

Из формулы (6.3) следует, что направление вектора импульса совпадает с направлением скорости. Единица импульса – килограмм-метр на секунду (кг×м/c).

Объединяя выражения (6.2) и (6.3), получаем

Полученное выражение позволяет предложить более общую формулировку второго закона Ньютона: действующая на тело сила равна производной импульса по времени .

Всякое действие тел друг на друга носит характер взаимодействия (рис. 6.1). В случае если тело действует на тело с некоторой силой , то и тело в свою очередь действует на тело с силой .

Третий закон Ньютона формулируется следующим образом: взаимодействующие тела действуют друг на друга с силами, равными по модулю и противоположными по направлению.

Эти силы, приложенные к разным телам, действуют по одной прямой и являются силами одной природы. Математическое выражение третьего закона Ньютона имеет вид

Знак “-” в формуле (6.5) означает, что векторы сил противоположны по направлению.

В формулировке самого Ньютона третий закон гласит: “Действию всœегда есть равное и противоположное противодействие, иначе – действия двух тел друг на друга между собою равны и направлены в противоположные стороны”.

Динамика – раздел механики, изучающий движение материальных тел совместно с физическими причинами, вызывающими это движение.

В основе динамики лежат законы Ньютона.

1. Закон инерции . Существуют такие СО, в которых всякое тело может находиться в состоянии покоя или равномерного прямолинейного движения, пока воздействие со стороны упругих сил не изменит его состояния.

Этот закон рассматривает тело как материальную точку и выполняется только в ИСО.

Сила – физическая величина, характеризующая воздействие на данное тело со стороны других тел, вызывающее изменение движения тела.

2. Закон движения материальной точки. Импульс тела . Скорость изменения импульса материальной точки равна действующей на неё силе F:

Изменение импульса в точке за время dt равно результирующей сил.

3. Закон взаимодействия. Если одно тело воздействует на другое с некоторой силой, то и второе воздействует на первое с той же силой.

Эти силы всегда одной природы, равны по модулю, противоположны по направлению и приложены к разным телам.

Динамика материальной точки. Основные уравнения движения материальной точки в дифференциальной форме.

Динамика системы частиц, центр инерции системы, закон движения центра инерции.

Рассмотрим систему точек с массами m1,m2…m n .

Центр масс – точка, для радиус-вектора которой выполняется:

Центр масс изолированной системы находится в состоянии покоя или равномерного прямолинейного движения.

Закон движения центра масс – в инерциальных системах отсчёта центр масс системы движется как материальная точка, в которой находится масса всей системы и на которую действует сила, равная геометрической сумме всех внешних сил, действующих на систему.

Динамика системы частиц, закон сохранения импульса в замкнутой системе.

В отсутствии сил импульс материальной точки остаётся неизменным по модулю и направлению (следствие из второго закона Ньютона).

Перепишем его для системы из N частиц:

где суммирование идет по всем силам, действующим на n-ю частицу со стороны m-ой. Согласно третьему закону Ньютона, силы вида и будут равны по абсолютному значению и противоположны по направлению, то есть Тогда после подстановки полученного результата в выражение (1) правая часть будет равна нулю, то есть:

Или

Как известно, если производная от некоторого выражения равна нулю, то это выражение есть постоянная величина относительно переменной дифференцирования, а значит:

(постоянный вектор).

То есть суммарный импульс системы частиц есть величина постоянная.

Как выглядит формула третьего закона Ньютона?


Как выглядит формула третьего закона Ньютона?

Третий закон Ньютона: определение и формула Определение его таково: силы, с которыми два тела действуют друг на друга, равны по величине и противоположны по направлению. Третий закон Ньютона можно записать в виде формулы: F_1 = – F_2, Где F_1 и F_2 силы действия друг на друга соответственно первого и второго тела.

Почему третий закон Ньютона называют законом взаимодействия?

Закон был впервые сформулирован И. Ньютоном в книге «Математические начала натуральной философии» (1687): Действию всегда есть равное и противоположное противодействие, иначе, взаимодействия двух тел друг на друга между собою равны и направлены в противоположные стороны.

Что гласит второй закон Ньютона?

Второй закон Ньютона в его наиболее распространённой формулировке, справедливой для скоростей, много меньших скорости света, утверждает: в инерциальных системах отсчёта ускорение, приобретаемое материальной точкой, прямо пропорционально вызывающей его силе, не зависит от её природы, совпадает с ней по направлению и . ..

Какая из формул выражает второй закон Ньютона?

В виде формулы в инерциальной системе отсчета выше сказанное запишем как: ¯F=m¯a(1), где m – масса тела; ¯F – сила, приложенная к телу и вызывающая его ускорение; ¯a – ускорение тела. Формула (1) – это второй закон Ньютона в классической динамике.

Как найти массу тела через второй закон Ньютона?

Измерение массы. нескольких тел, а затем соединить все эти тела вместе и измерить массу m одного объединенного тела, то будет выполняться простое соотношение: m=m1+m2+m3+ … . Справедливо и обратное: если разделить тело на части, то сумма масс этих частей будет равна массе тела до разделения.

Какой формулой описывается второй закон динамики?

Данная формулировка справедлива для систем с постоянной массой. В случае, когда масса тела изменяется (например, при релятивистском движении), второй закон Ньютона имеет вид F=dpdt, где p − импульс (количество движения) тела. В общем случае сила F зависит от координат тела, т. е.

Что такое 4 закон Ньютона?

Четвертый закон Ньютона ещё называют законом независимости действия сил. … Материальная точка под действием нескольких сил приобретает ускорение, равное геометрической сумме ускорений, которые она получила бы от каждой силы, действующей отдельно, независимо от других.

Для чего нужен 1 закон Ньютона?

Таким образом, во взлетающем корабле первый закон Ньютона не выполняется. … Именно для того и нужен первый его закон, чтобы определить, можно ли в данной системе отсчета пользоваться остальными законами Ньютона. Системы отсчета, в которых выполняется первый закон Ньютона, называют инерциальными системами отсчета.

Что объясняет третий закон Ньютона?

Это равенство называется третьим законом Ньютона. … Тела действуют друг на друга с силами, равными по модулю и противоположными по направлению. Силы, возникающие при взаимодействии тел, всегда имеют одинаковую природу.

Чем прославился и Ньютон?

Английский математик, астроном, физик, механик, заложивший основы классической механики, один из создателей классической физики – Ньютон объяснил движение небесных тел – планет вокруг Солнца и Луны вокруг Земли. Самым известным его открытием был закон всемирного тяготения. Сэр Исаак Ньютон (англ.

Какие открытия сделал Ньютон?

Автор фундаментального труда «Математические начала натуральной философии», в котором он изложил закон всемирного тяготения и три закона механики, ставшие основой классической механики.

Что открыл Ньютон после того как на него упало яблоко?

Ньютон открыл свой знаменитый закон всемирного тяготения после того, как ему на голову упало яблоко – это знают многие. … Стакли описал, как в 1726 году они с Ньютоном пили чай под яблоней. И Ньютон припомнил, что в такой же обстановке открыл закон притяжения.

Что изобретал Ньютон?

Исаак Ньютон изложил закон всемирного тяготения и три закона механики, ставшие основой классической механики. Он дал теорию движения небесных тел, создав основы небесной механики.

Какой вклад в науку внес Ньютон?

Исаак Ньютон (1643-1727) – выдающийся английский физик, механик, астроном и математик – сформулировал основные законы классической механики, открыл закон всемирного тяготения, закон разложения белого света на монохроматические составляющие, разработал (наряду с Лейбницем) дифференциальное и интегральное исчисление.

Кто такой Ньютон кратко?

Исаак Ньютон (1642–1727 гг.) – выдающийся английский ученый, один из создателей классической физики. Биография Ньютона богата во всех смыслах этого слова. Он сделал немало открытий в области физики, астрономии, механики и математики, в том числе открыл закон всемирного тяготения.

Как умер Ньютон?

31 марта 1727 г.

В каком году родился Ньютон?

4 января 1643 г.

Как правильно ставить ударение Ньютон?

В слове “ньЮтон” ударение ставится на гласный “ю” – ньЮтон. Слово происходит от имени собственного иноязычного происхождения – НьЮтон. Ударение в слове “ньЮтон” сохраняет звучание существительного в английском оригинале при переводе его на русский язык. Ударение падает на первый слог.

Где умер Исаак Ньютон?

Кенсингтон

Где учился Ньютон?

Тринити-колледж1667 г.–1668 г.

Что такое единица Ньютон?

Ньютон — производная единица. Исходя из второго закона Ньютона она определяется как сила, изменяющая за 1 секунду скорость тела массой 1 кг на 1 м/с в направлении действия силы. Таким образом, 1 Н = 1 кг·м/с2.

Что сказал Ньютон?

Исаак Ньютон прославился тем, что, когда яблоко упало ему на голову, он единственный, кто задумался, а не просто вскрикнул: “Твою мать!”. … Падает на голову яблоко.

Что было написано на могиле у Ньютона?

На могиле Ньютона оставлена надпись: «Здесь покоится сэр Исаак Ньютон, дворянин, который почти божественным разумом первый доказал с факелом математики движение планет, пути комет и приливы океанов. Он исследовал различие световых лучей и появляющиеся при этом различные свойства цветов, чего ранее никто не подозревал.

Кто придумал закон Ньютона?

Исааком Ньютоном

Кому яблоко упало на голову?

Все мы с детства помним рассказ про Исаака Ньютона и упавшее ему на голову яблоко. Якобы именно этот случай помог великому ученому сформулировать закон всемирного тяготения.

Как Ньютон доказал закон всемирного тяготения?

Закон всемирного тяготения был сформулирован Исааком Ньютоном (1643−1727) и опубликован в 1687 году. В соответствии с этим законом, два точечных тела притягиваются друг к другу с силой, которая прямо пропорциональна массам этих тел m1 и m2 и обратно пропорциональна квадрату расстояния между ними: F=Gm1m2r2.

Как звучит первый закон Ньютона?

Законы Ньютона (основная школа) Первый закон Ньютона. Если на тело не действуют силы или их действие скомпенсировано, то данное тело находится в состоянии покоя или равномерного прямолинейного движения. … Если два тела взаимодействуют друг с другом, то ускорения этих тел обратно пропорциональны их массам.

Кто придумал второй закон Ньютона?

Но при этом Гук сформулировал его раньше Ньютона. И действительно, три величины: сила, масса и ускорение связаны между собой основным законом механики–вторым законом Ньютона–в следующей всеобъемлющей формуле: приложенная сила равна массе, умноженной на ускорение.

Видео с вопросами: Использование первого закона Ньютона для расчета сил

Стенограмма видео

Корабль массой 7000 ньютонов имеет мотор и паруса. Корабль движется с постоянной скоростью по поверхности моря. Мотор обеспечивает усилие в 8500 ньютонов, а сопротивление воды вокруг корабля создает усилие в 6000 ньютонов. Ветер дует в направлении, противоположном движению корабля, поэтому паруса создают силу лобового сопротивления корабля. С какой силой действует на корабль ветер, дающий паруса? С какой силой вода под кораблем действует вверх на корпус корабля?

Итак, это вопрос, состоящий из двух частей, о различных силах, действующих на корабль. Давайте начнем с рисования диаграммы, показывающей, что представляют собой все эти силы. Вот это наш корабль, и нам в вопросе говорят, что он имеет вес 7000 ньютонов. Эта сила веса возникает в результате силы тяжести и действует вертикально вниз. Обозначим эту силу как 𝐹 с индексом 𝑔, где 𝑔 означает гравитацию. Здесь следует вспомнить, что сила является векторной величиной. И важно помнить о векторных величинах, что они имеют не только величину, но и направление.

Мы указали направление силы веса с помощью стрелки, указывающей вниз на нашей диаграмме. Поскольку у сил есть направление, а также величина, это означает, что нам нужно выбрать направление, которое будет считаться положительным. Когда мы говорим о вертикальных силах, принято считать, что положительное направление направлено вверх. При положительном направлении вверх сила веса корабля, индекс 𝐹, индекс 𝑔, равна отрицательным 7000 ньютонам, где отрицательный знак означает, что эта сила действует вертикально вниз в отрицательном направлении.

Мы также должны выбрать горизонтальное направление, чтобы считать его положительным. А для этого имеет смысл принять положительное направление как направление движения корабля. Давайте представим, что корабль движется в этом направлении вправо, поэтому мы примем право за положительное направление. Другими словами, силы, действующие вправо, положительны, а силы, действующие влево, отрицательны.

Вопрос говорит нам о том, что двигатель обеспечивает силу 8500 ньютонов. Поскольку двигатель обеспечивает движение корабля, то направление силы, обеспечиваемой двигателем, должно совпадать с направлением движения корабля. Так это справа. Мы обозначим эту силу индексом 𝐹 𝑚. И мы знаем, что индекс 𝐹 𝑚 равен 8500 ньютонов, где значение положительно, потому что сила действует вправо.

Нам также сообщили, что сила сопротивления воды вокруг корабля составляет 6000 ньютонов. Эта сила сопротивления является результатом сопротивления воды движению корабля. Таким образом, сила сопротивления должна действовать влево в направлении, противоположном движению корабля. Обозначим эту силу сопротивления воды индексом 𝐹 𝑑. У нас есть индекс 𝐹 𝑑, равный минус 6000 ньютонов, где он отрицательный, потому что действует слева.

Эти три силы — все те силы, значения которых нам известны. Однако есть еще две силы, которые нам нужно добавить к нашей диаграмме. Вопрос говорит нам, что есть вторая сила сопротивления, обеспечиваемая парусами, потому что ветер дует в направлении, противоположном движению корабля. Из этого можно сделать вывод, что сила, создаваемая парусами, должна действовать в направлении, противоположном движению корабля. Итак, это слева на нашей диаграмме. Обозначим эту силу индексом 𝐹 𝑠. А значение индекса 𝐹 𝑠 — это как раз то, что нас просят найти в первой части вопроса.

Последняя сила, которую нам нужно добавить к нашей диаграмме, — это сила, упомянутая во второй части вопроса. Это восходящая сила реакции, действующая на корабль со стороны воды. Итак, давайте добавим эту направленную вверх силу реакции на нашу диаграмму и обозначим ее индексом 𝐹 𝑟.

Итак, у нас есть диаграмма, показывающая все силы, действующие на корабль. Следующим шагом будет выяснить, как мы можем использовать эту информацию, чтобы найти значения сил, которые мы в настоящее время не знаем. Для этого воспользуемся первым законом движения Ньютона. Этот закон гласит, что объект, находящийся в состоянии покоя, остается в покое, а объект, движущийся с постоянной скоростью, продолжает двигаться с этой скоростью, если на него не действует неуравновешенная сила.

Первый закон движения Ньютона говорит нам, что если на объект не действует результирующая сила, то скорость этого объекта не изменится. Мы также можем перевернуть эту логику и сказать, что если скорость объекта не меняется, то на него не должно действовать результирующая сила.

Текст вопроса говорит нам, что корабль движется по морю с постоянной скоростью. Так как корабль движется в заданном направлении с постоянной скоростью, то и скорость его должна быть постоянной. Поскольку скорость корабля не меняется, первый закон движения Ньютона означает, что на него не должно действовать результирующая сила. На практике это означает, что три горизонтальные силы, действующие на корабль, должны уравновешивать друг друга, а две вертикальные силы также должны уравновешивать друг друга.

Давайте немного освободимся и посмотрим, как мы можем использовать эту информацию, чтобы ответить на две части вопроса.

Первая часть вопроса касается силы, действующей на корабль от ветра, толкающего его паруса. Таким образом, нас просят найти значение горизонтальной силы сопротивления, которую мы обозначили индексом 𝐹 𝑠. Из первого закона движения Ньютона мы пришли к выводу, что на корабль не должна действовать результирующая сила, а это также означает, что на него не должно действовать результирующая горизонтальная сила. Отсутствие суммарной горизонтальной силы означает, что сумма всех горизонтальных сил, действующих на судно, должна быть равна нулю.

Три горизонтальные силы: 𝐹, нижний индекс 𝑚, сила, обеспечиваемая двигателем, 𝐹, нижний индекс 𝑑, сила сопротивления воды, и 𝐹, нижний индекс 𝑠, сила сопротивления ветра, дающего паруса. Это означает, что у нас 𝐹 нижний индекс 𝑚 плюс 𝐹 нижний индекс 𝑑 плюс 𝐹 нижний индекс 𝑠 равен нулю ньютонов.

Мы пытаемся найти значение индекса 𝐹 𝑠. Поэтому нам нужно изменить уравнение, чтобы 𝐹 нижний индекс 𝑠 стал предметом. Для этого мы вычитаем нижний индекс 𝐹 𝑚 и нижний индекс 𝐹 𝑑 из обеих частей уравнения. В левой части положительные и отрицательные термины 𝐹 нижнего индекса 𝑚 и 𝐹 нижнего индекса 𝑑 компенсируют друг друга. Остается только термин 𝐹 Подстрочный 𝑠. В правой части вычитание этих двух членов из нуля ньютонов просто дает нам отрицательный индекс 𝐹 𝑚 минус индекс 𝐹 𝑑.

Теперь, когда у нас есть уравнение, где 𝐹 нижний индекс 𝑠 является предметом, мы готовы подставить наши значения для 𝐹 нижнего индекса 𝑚 и 𝐹 нижнего индекса 𝑑. Когда мы это сделаем, нам нужно будет позаботиться о негативных знаках. Подставляя значения, мы получаем, что индекс 𝐹 𝑠 равен минус 8500 ньютонов минус минус 6000 ньютонов. Эти два отрицательных знака в конечном итоге отрицают друг друга, давая нам плюс. Таким образом, у нас есть индекс 𝐹 𝑠, равный минус 8500 ньютонов плюс 6000 ньютонов, что получается как минус 2500 ньютонов.

Теперь понятно, что эта сила отрицательна, потому что на нашей диаграмме она действует влево, что, как мы знаем, является отрицательным направлением. И это значение индекса 𝐹 𝑠 представляет собой силу, действующую на корабль от ветра, толкающего его паруса, что и требовалось найти в первой части вопроса. Итак, наш ответ на этот первый вопрос — отрицательные 2500 ньютонов.

Хорошо, давайте немного освободимся, чтобы рассмотреть вторую часть вопроса.

Эта вторая часть запрашивает у нас восходящую силу реакции на корабле. Так что это значение количества, которое мы пометили как 𝐹 нижним индексом 𝑟 на нашей диаграмме. В первой части вопроса мы использовали первый закон движения Ньютона, чтобы вывести, что на корабль не должно действовать результирующая горизонтальная сила. Этот же закон также говорит нам, что на него не должна действовать результирующая вертикальная сила. В этом случае действуют только две вертикальные силы. Есть индекс 𝐹 𝑟, сила реакции воды, и индекс 𝐹 𝑔, сила веса.

Так как мы знаем, что результирующая вертикальная сила отсутствует, то сумма этих двух сил, индекс 𝐹 𝑟 плюс индекс 𝐹 𝑔, должна быть равна нулю ньютонов. Мы пытаемся найти значение индекса 𝐹 𝑟. Итак, давайте сделаем это предметом уравнения. Мы можем сделать это, вычитая нижний индекс 𝐹 𝑔 с обеих сторон. В правой части положительные и отрицательные термины 𝐹 нижнего индекса 𝑔 компенсируют друг друга. Таким образом, мы приходим к уравнению, которое говорит, что индекс 𝐹 𝑟 равен отрицательному индексу 𝐹 𝑔.

Теперь, когда у нас есть это уравнение для силы реакции, 𝐹, индекс 𝑟, все, что нам нужно сделать, это добавить наше значение для силы веса, 𝐹, индекс 𝑔. Это дает нам, что индекс 𝐹 𝑟 равен отрицательным минусам 7000 ньютонов. Два отрицательных знака компенсируют друг друга, что дает нам положительный результат в 7000 ньютонов, что имеет смысл, поскольку сила реакции направлена ​​вверх, что мы приняли за положительное. Эта направленная вверх реактивная сила, прикладываемая водой к кораблю, и есть то, что нас просили найти. Итак, наш ответ на вторую часть вопроса — 7000 ньютонов.

ньютоновская механика – Как первый закон Ньютона совместим с уравнениями переноса?

$\begingroup$

Итак, я изучаю неравновесную термодинамику и первое, что я усвоил, это то, что эта дисциплина имеет первую область обобщения идеи уравнений переноса.

Уравнения переноса обычно имеют вид: $$J=-A\cdot\frac{dX}{dx}$$

Где $J$ — поток чего-то, движущегося по площади, $X$ — некоторая величина, определенная в каждой части физической системы (вязкость , масса, температура и т. д.), а $A$ — константа пропорциональности. $\frac{dX}{dx}$ можно рассматривать как градиент этой величины $X$ и, таким образом, как своего рода силу.

Существует несколько уравнений переноса, связанных с различными явлениями. Например, закон Фурье представляет собой уравнение переноса, которое связывает градиент температуры с тепловым потоком, причем градиент температуры является движущей силой этого потока энергии между различными частями системы.

Итак, вывод здесь (по крайней мере, так говорит мой учитель) состоит в том, что не может быть потока любой величины без какого-либо градиента (движущей силы) и наоборот .

Но это, даже если это звучит разумно, противоречит некоторым моим представлениям о первом законе движения Ньютона: если у меня есть твердый стержень, движущийся в пространстве с равномерным линейным движением, то я могу заключить, что сила, толкающая его, отсутствует. Стержень движется только потому, что у него есть инерция, и, следовательно, он будет продолжать двигаться, если к смеси не будет добавлена ​​​​сила. И если я представлю вычисление потока массы внутри круглой области, когда стержень пересекает ее, то я увижу, что существует какой-то постоянный поток материи, когда стержень проходит через нее. Получается, что мы имеем ситуацию, когда есть поток, но нет движущей силы, есть только инерция. Таким образом, утверждение о том, что потоки и потоки возникают из-за неравномерности некоторой величины в пространстве, перестает быть правдой.

Мой вопрос: где я ввел вас в заблуждение? Дело в том, что стержень, который я описываю, не является термодинамической системой, подверженной огромному количеству взаимодействий, а может рассматриваться как микроскопическая система (состоящая всего из одного элемента), и поэтому уравнения переноса здесь неуместны? Может быть, поток массы связан не с этой конкретной силой, а с другой общей силой, о которой я не говорю? Не следует ли здесь почему-то смешивать движущую силу как обобщенное понятие с собственно механической силой? Что именно мне не хватает?

  • ньютоновская механика
  • термодинамика
  • энтропия
  • неравновесность

$\endgroup$

6

$\begingroup$

Для того, чтобы иметь (постоянный, ненулевой) поток, вам нужно либо вообще отсутствие силы, либо баланс сил. Обычно здесь обсуждаются ситуации, когда имеется либо трение, либо сила, подобная трению, которая противодействует движению (и, следовательно, потоку). В этом случае вам понадобится какая-то другая сила, например, результат градиента, чтобы преодолеть трение. Коэффициенты часто называют такими именами, как «проводимость», что является хорошим названием, но имя может заставить неосторожных забыть тот факт, что это эффект, подобный трению.

Общее трение или удельное сопротивление обратно пропорционально общей проводимости. Итак, если нет трения, как в случае идеализированной ситуации, обсуждаемой в первом законе Ньютона, то «проводимость» бесконечна.

В классической физике, я думаю, было бы правильно сказать, что «всегда» нужен какой-то градиент, чтобы иметь поток, если кто-то говорит, что идеализация полного отсутствия трения — это всего лишь идеализация. Например, идея Ньютона о теле, движущемся в пустом пространстве, является идеализацией в том смысле, что пространство никогда не бывает полностью пустым (ни один вакуумный насос не создаст идеальный вакуум). Но в таких явлениях, как сверхпроводимость и сверхтекучесть, вы можете иметь постоянный ток, который никогда не затухает. И токи такого рода можно обнаружить и внутри электронной структуры каждого атома.

$\endgroup$

$\begingroup$

Мне сложно ответить на Ваш вопрос словами.

В вашем примере с движущимся стержнем вы предположили, что нет сил, противодействующих движению стержня (например, трения), и поэтому «феноменологическая постоянная» в вашем примере бесконечна.
Таким образом, для переноса массы не требуется перепада давления (силы), точно так же, как не требуется разности потенциалов для переноса электронов (электрического тока) в сверхпроводнике.

$\endgroup$

0

$\begingroup$

Предполагается, что потоки в вашем уравнении относятся к наблюдателю, движущемуся со средней скоростью (см. «Феномен переноса», Берд и др.). Таким образом, для вашего стержня, движущегося с постоянной осевой скоростью v в направлении x, общий поток тепла равен $$\Phi=\rho CvT-k\frac{dT}{dx}$$, где C – теплоемкость стержня. стержень.

$\endgroup$

$\begingroup$

В случае со стержнем вы можете найти систему отсчета, в которой никакая масса не проходит через эту область. Нет никакой силы, тянущей частицы через поверхность. Концентрация частиц в вашем стержне постоянна по всему стержню. Градиент нулевой. В остальной части стержня чистого движения нет.

Если вы посмотрите на законы Фика, вы можете добавить дополнительное смещение массы, вызванное движением всей вашей системы. В каждом кадре есть градиент, на который не влияет глобальная скорость. Градиент «силы» является абсолютным. Присутствует в каждом кадре. Если вы сравните ситуации в системе покоя, для стержня и вашей системы градиента концентрации, вы увидите, что в этом нет ничего плохого.

$\endgroup$

$\begingroup$

TL; DR: микроскопическое и макроскопическое движение

Разница здесь сродни разнице между теплом и работой : одно представляет собой перенос энергии на микроскопическом уровне, тогда как другое представляет собой макроскопическое движение тех же молекул, что и целое. Другая распространенная пара микроскопический/макроскопический — это диффузия и конвекция, когда мы говорим о переносе массы. Точно так же можно найти подходящие пары для любого из явлений переноса в таблице, приведенной в ОП.

Микроскопическое движение не нарушает законов Ньютона, но эти законы должны применяться на молекулярном уровне с учетом взаимодействия между молекулами. С другой стороны, пример движущегося стержня представляет собой макроскопическое движение молекул стержня вместе, характеризующееся движением центра масс.

Еще одна тонкость заключается в том, что термодинамические законы обычно выводятся и обсуждаются в системе отсчета, в которой тело не движется и не вращается, — они не исключают движения тела (например, резервуара с газом) в целом, которое было бы происходят в соответствии с законами Ньютона.

$\endgroup$

$\begingroup$

Итак, вывод здесь (по крайней мере, так говорит мой учитель) заключается в том, что не может быть потоком любой величины без какого-либо градиента (а движущая сила) и наоборот.

Этот вывод правильный, и я не вижу, как это должно противоречить законам Ньютона. Я бы воздержался от того, чтобы называть это движущей силой , потому что это может быть аналогия, но, возможно, не самая лучшая.

Представьте, что в своем мысленном эксперименте с стержнем вы окружили проблему большим ящиком с перегородкой посередине. Ваш стержень находится на левой стороне, правая сторона пуста. Существует градиент концентрации (стержни 1 и 0), и когда стержень проходит через разделитель, вы можете приписать поток этому транспортному явлению.

Теперь этот поток будет дискретным, но ничего не изменив, кроме количества частиц, вы получите классический диффузионный эксперимент. Каждая частица не будет ускоряться никакими силами, для моделирования процесса достаточно упругих столкновений со стенками и другими частицами.

$\endgroup$

Твой ответ

Зарегистрируйтесь или войдите в систему

Зарегистрируйтесь с помощью Google

Зарегистрироваться через Facebook

Зарегистрируйтесь, используя адрес электронной почты и пароль

Опубликовать как гость

Электронная почта

Требуется, но никогда не отображается

Опубликовать как гость

Электронная почта

Требуется, но не отображается

Нажимая «Опубликовать свой ответ», вы соглашаетесь с нашими условиями обслуживания, политикой конфиденциальности и политикой использования файлов cookie

.

Закон охлаждения Ньютона | Центр знаний Каролины

Предположим, вы пытаетесь охладить напиток. Остынет ли купленная вами газировка комнатной температуры к вечеринке? Теперь вы можете рассчитать, сколько времени понадобится напитку, чтобы достичь температуры холодильника. Все, что вам нужно сделать, это применить закон охлаждения Ньютона.

Закон охлаждения Ньютона гласит, что скорость охлаждения объекта пропорциональна разнице температур между объектом и окружающей средой. Проще говоря, стакан горячей воды быстрее остынет в холодной комнате, чем в жаркой. Этот простой принцип относительно легко доказать, и эксперимент дает повторяемые и воспроизводимые результаты. Этот эксперимент также является прекрасной возможностью для междисциплинарного урока, включающего физику и курсы продвинутой математики, такие как алгебра II, предварительное исчисление и исчисление.

Термодинамика

Предыстория

Перед выполнением этих заданий учащимся потребуется некоторая базовая справочная информация по термодинамике. Учащиеся должны быть знакомы с первым и вторым законами термодинамики .

первый закон термодинамики  по сути является законом сохранения энергии. Это означает, что энергия может изменять форму. Горячая вода, которую вы используете для этого эксперимента, содержит тепло или тепловую энергию. Когда вы использовали плиту, микроволновую печь или плиту для нагрева воды, вы преобразовывали электрическую энергию в тепловую. Общее количество энергии во Вселенной постоянно. Энергия может менять форму, но общее количество остается прежним. Энергия сохраняется.

второй закон термодинамики  утверждает, что энтропия или беспорядок во Вселенной всегда увеличивается. Для целей данного эксперимента это означает, что тепло всегда переходит от горячего объекта к холодному. Если ваш суп слишком горячий и вы добавляете немного льда, чтобы охладить суп, охлаждения не происходит, потому что «холод» переходит от льда к супу. Скорее, тепло от супа растапливает лед, а затем уходит в атмосферу. В этом эксперименте тепло от горячей воды передается воздуху, окружающему стакан с горячей водой.

Существует три способа передачи тепла.

  • Проводимость  возникает при прямом контакте. Прикоснитесь к горячей плите, и тепло будет передано вашей руке.
  • Конвекция  возникает при массовом движении жидкости (жидкость означает жидкость или газ). Используйте вентилятор для охлаждения, и тепло передается от вас к окружающему воздуху за счет конвекции.
  • Излучение  – передача тепла в виде волн. Встаньте на солнечный свет, и вы почувствуете тепло, передаваемое от солнца излучением.

Закон охлаждения Ньютона применяется к конвективному теплообмену; это не относится к тепловому излучению.

Закон охлаждения Ньютона гласит, что скорость теплообмена между объектом и окружающей его средой пропорциональна разнице температур между объектом и окружающей средой. Математически это представляется как:

Это также может быть выражено следующим уравнением:

У этого уравнения есть 2 общих решения. Если температура объекта T выше температуры окружающей среды T a , тогда:

Уравнение 1:

Если температура окружающей среды, T a , меньше температуры объекта, T, решение уравнения:

Уравнение 2:

Решение к дифференциальному уравнению дает две экспоненциальные функции, которые можно использовать для прогнозирования будущей температуры охлаждаемого объекта в данный момент времени или времени, в течение которого объект охлаждается до заданной температуры.

В этом эксперименте стакан с горячей водой будет охлаждаться до температуры окружающей среды, и будет использоваться следующее уравнение:

Materials

  • Water
  • Beaker, 250 mL
  • Thermometer
  • Hot Plate
  • Ice Bath or Refrigerator
  • Scientific Calculator
  • Graph Paper or Computer with Spreadsheet Software

Safety

Observe all standard lab safety процедуры и протоколы. Носите соответствующие средства индивидуальной защиты (СИЗ). Эта лабораторная работа включает в себя использование горячей плиты и горячей воды. Надевайте защитные очки при нагревании и перемещении горячей воды и используйте щипцы или термостойкие перчатки для перемещения горячей мензурки. Выключите и отсоедините конфорку, когда нагрев завершится, и не забывайте всегда обращаться с поверхностью конфорки так, как если бы она была горячей.

Процедура

Мероприятие 1: График и анализ данных для охлаждающей воды

  1. Используйте термометр для измерения температуры атмосферы. Запишите эту информацию как T a  в Таблицу 1. ( Примечание:   В качестве альтернативы для сбора данных можно использовать систему программного обеспечения с датчиком температуры.)
  2. Нагрейте примерно 200 мл воды в стакане. Установите стакан на лабораторный стол, изолированный от поверхности стола, где его никто не будет беспокоить.
  3. С помощью термометра запишите температуру горячей воды. Запишите это значение как T(0) в Таблицу 1.
  4. Запустите таймер и продолжайте записывать температуру каждые 10 минут. Запишите эти данные в Таблицу 1.

Повторите процедуру, измерив температуру снаружи, в ледяной бане или в холодильнике для T a . Используйте тот же объем горячей воды, начиная с той же температуры. Запишите данные в Таблицу 1.

Постройте график и сравните свои результаты. График температуры по оси y и времени по оси x.

Если вы используете электронную таблицу для построения графика данных и добавляете линию тренда, выберите «экспоненциальную функцию».

Примечание:  При необходимости конвертируйте из °F в °C. ° C = (5/9) (° F – 32)

Температура (° C) Температура (° C) Время (минуты)

13
(минуты)

3

(минуты)

3

(Министер Т(0) Т(0) 0
Т(10) Т(10) 10
T(20) T(20) 20
T(30) T(30) 30
T(40) T(40) 40
T(50) T(50) 50
T(60) T(60) 60
Table 1

Questions for Activity 1

  1. Чем отличается линия, обозначающая водяное охлаждение в классе и водяное охлаждение в холодильнике/снаружи?
  2. Как график говорит нам, верна наша гипотеза или нет?
  3. Что является независимой переменной в этом эксперименте?
  4. Что является зависимой переменной в этом эксперименте?
  5. Какие элементы управления использовались в этом эксперименте?
  6. Какие еще факторы могли повлиять на результаты этого эксперимента?

Образцы данных и ответы

Образцы данных Задание 1

Ответы к заданию 1

  1. Поскольку линия на графике идет слева направо, температура должна снижаться. Если закон охлаждения Ньютона верен, линия, изображающая более холодную атмосферу, должна уменьшаться быстрее.
  2.  Данные показывают, что образец воды, находящийся в атмосфере с более низкой температурой, охлаждается быстрее. Это согласуется с законом охлаждения Ньютона.
  3. Независимой переменной является температура.
  4. Зависимой переменной является время.
  5. Некоторыми элементами контроля могут быть: вещество (вода), масса вещества (200 мл = 200 г воды), контейнер, температура атмосферы, стабильная атмосфера (отсутствие изменений температуры или конвекционных потоков от вентилятора). или открытое окно).
  6. Факторы, которые могут быть изменены, включают: запуск при более высокой или более низкой температуре, использование другой массы воды, использование другого контейнера (например, термоса или чашки из пеноматериала) или использование другого вещества (например, раствора сахара). или тарелка супа). Вы также можете провести эксперимент с холодной жидкостью и горячей атмосферой, например, согревшись стаканом холодной воды в жаркий день.

Упражнение 2: Работа с уравнением закона охлаждения Ньютона

Это упражнение представляет собой математическое упражнение. Применить Уравнение 2  к данным, собранным в Упражнении 1, для прогнозирования температуры воды в заданное время.

Когда температура воды или вещества, которое охлаждается, T, больше, чем температура окружающей атмосферы T a ¸ решение этого уравнения:

Температура как функция времени зависит от переменных C 2 , к и Т а . Если эти значения известны, то температуру t в любое время можно найти, просто подставив это время вместо t в уравнение. Или время, за которое объект достигает определенной температуры, можно найти, найдя t и подставив T (t) вместо заданной температуры.

Рассмотрим следующий набор данных для образца воды объемом 200 мл, который охлаждается в течение часа.

Temperature (°C) Temperature (°C) Time (minutes)
T a    25
T(0 ) 72 0
Т(10) 59 10
Т(20) 52 20
T(30) 46 30
T(40) 40 40
T(50) 37 50
T(60) 29 60
Таблица 2

Найдите C 2 , подставив данные о времени и температуре вместо T(0).

При t = 0 температура равна 72. При t = 0 e -kt становится равным 1.

Теперь используйте другую точку данных, чтобы найти значение k.

Чтобы найти значение k, возьмите натуральный логарифм обеих сторон:

Теперь используйте эти две константы, чтобы предсказать температуру в какой-то момент в будущем, и используйте данные в таблице 1, чтобы проверить ответ.

Попробуйте найти температуру в момент времени t = 40 минут.

Используйте калькулятор, чтобы найти значение:

Это близко к образцу даты в Таблице 2.

Теперь попробуйте предсказать, сколько времени потребуется, чтобы температура достигла 30°.

Взятие натурального логарифма обеих сторон:

Решение для t:

Детали для вывода уравнений 1 и 2

Учащиеся, имеющие некоторый опыт в вычислениях, могут захотеть узнать, как вывести уравнения 1 и 2.

Приступить к решению дифференциальное уравнение путем перестановки уравнения:

Интегрируем обе части:

По определению это означает:

Используя законы показателей степени, это уравнение можно записать как:

Количество e C1  является константой, которую можно выразить как C 2 .

У этого уравнения есть 2 общих решения. Одно из решений состоит в том, что вещество при температуре T горячее, чем температура окружающей среды T a . Поскольку выражение в левой части уравнения находится между столбцами абсолютного значения, (T – T a ) может быть как положительным, так и отрицательным.

Решения, как указано ранее, даются:

Уравнение 1:

Уравнение 2:

Уравнение 1 применяется, если температура объекта или вещества T выше температуры окружающей среды T a ; Уравнение 2 применяется, если температура окружающей среды выше температуры объекта или вещества.

Что делать, если температура атмосферы теплее, чем образец вещества? Как долго стакан лимонада будет оставаться холодным в летний день?

В случае, если атмосфера теплее вашего материала, решение закона охлаждения Ньютона выглядит так:

Можете ли вы разработать процедуру проверки этого уравнения? Начните с образца холодной воды и повторите процесс, описанный в Упражнении 2. Попробуйте предсказать, сколько времени потребуется, чтобы вода достигла комнатной температуры.

Магазин тепла и термодинамики

Курсы физики в старших классах обычно начинаются с изучения классической механики. В начале курса студенты знакомятся с уравнениями движения, уравнениями кинематики. Кинематика Кинематика — это изучение движения объектов без учета сил, вызывающих движение. Эти знакомые уравнения позволяют учащимся анализировать и прогнозировать […]

Это предварительное лабораторное исследование, основанное на уроке творческой статистики, проведенном ведущим преподавателем естественных наук Марком Кротеком, знакомит с ценностью статистики, неотъемлемыми качествами неопределенности и интерпретацией данных. Студентов просят опровергнуть утверждение о том, что их учитель обладает экстрасенсорными способностями, используя хитрый карточный фокус, тщательный протокол планирования эксперимента и применение теста хи-квадрат.

Обнаружение ИК-излучения с помощью смартфона Солнце на самом деле излучает больше света, чем та его часть, которую мы видим. Сразу за видимым фиолетовым светом находится ультрафиолетовый (УФ) свет, а сразу за видимым красным светом — инфракрасный (ИК) свет. Ни тот, ни другой не видны человеческому глазу. Инфракрасный свет не зависит от видимого света — инфракрасный свет может […]

Законы Ньютона: объяснение, обзор и примеры

Исаак Ньютон изучал физику сил — как они могут быть созданы, как они взаимодействуют друг с другом и как они создают мир, который мы наблюдаем каждый день. Хотя Ньютон, возможно, и не дал точного определения того, как все работает на всех масштабах Вселенной, законы Ньютона до сих пор почитаются и изучаются. В этой части вы узнаете, что это за законы, увидите некоторые примеры законов Ньютона и поймете, почему они по-прежнему так важны сегодня.

Заинтересованы в лицензии школы Альберта?

В истории было несколько случаев, когда новое научное открытие потрясало наше фундаментальное понимание того, как функционирует Вселенная в целом. Открытие законов Ньютона — одно из них. До Ньютона ученые считали, что у объектов есть неотъемлемое количество энергии для движения, и что вещи перестают двигаться, когда у них заканчивается энергия. Хотя на первый взгляд это может иметь некоторый смысл, Ньютон смотрел дальше этого. Он осмелился задаться вопросом, что произойдет с мячом, который катится, не встречая трения, и не накладывает на него биологических ограничений.

Что мы рассматриваем

Первый закон движения Ньютона
Первый закон Ньютона гласит, что объект сохраняет постоянную скорость и направление, если на него не воздействуют внешние силы.

Можно также сказать, что объект будет поддерживать постоянную скорость, если на него не воздействуют внешние силы, поскольку скорость учитывает как скорость, так и направление. Стоит отметить, что это включает в себя скорость 0\text{ м/с}, так как неподвижный объект не начнет двигаться, если внешняя сила не заставит его двигаться.

Если вам не совсем нравится идея «внешней силы», это означает, что сила исходит от чего-то вне объекта. Например, если вы встанете позади автомобиля, застрявшего на льду, и толкнете его, он в конечном итоге начнет движение, если вы сможете приложить достаточное усилие. Однако если бы вы сели в машину и нажали на приборную панель, она бы не двигалась. Разница здесь в том, что в первый раз вы прикладываете силу снаружи автомобиля, а во второй раз вы применяете силу внутри автомобиля. Внешние силы также могут иногда называться внешними силами.

Чистая сила означает ненулевую силу, которая не уравновешивается другими силами. Сила представляет собой вектор с величиной и направлением. Когда есть сила 5\text{ N}, действующая вправо, и сила 5\text{ N}, действующая влево, эти две силы уравновешиваются, и результирующая сила равна нулю. Две силы уравновешены.

Возможно, вы слышали, что это перефразировано по-другому: « тело в движении имеет тенденцию оставаться в движении, в то время как тело в состоянии покоя стремится оставаться в покое» . Хотя это относительно точная интерпретация, она также упрощена и не объясняет, как тело будет продолжать двигаться, как это делают наши исходные два утверждения.

Как по-другому называется Первый закон движения Ньютона?

Подобно тому, как мы можем переформулировать Первый закон Ньютона разными способами, его также можно назвать по-разному. Другое название первого закона Ньютона — закон инерции . Инерция — это мера того, насколько сложно изменить движение объекта. Первый закон Ньютона говорит нам, что только внешняя сила может изменить движение объекта. Называть это законом инерции имеет смысл, поскольку оба относятся к объекту, которому требуется усилие из внешнего источника для изменения скорости.

Заинтересованы в лицензии школы Альберта?

Примеры законов Ньютона: первый закон движения Ньютона

Давайте начнем с некоторых примеров законов Ньютона, начиная с первого закона движения Ньютона. Мы начнем со знакомых повседневных переживаний, а затем перейдем к менее распространенным.

Пример 1. Мяч на игровом поле

Игрок бьет по мячу на ровном игровом поле в нормальных земных условиях. Если предположить, что мяч ни во что не врежется и никто его не коснется, что произойдет с мячом?

  1. Мяч остановится и изменит направление.
  2. Мяч не остановится, а изменит направление.
  3. Мяч остановится, но не изменит направление.
  4. Мяч не остановится и не изменит направление.

У вас может возникнуть соблазн сказать 4 , потому что мы только что узнали о первом законе Ньютона. Однако помните, мы говорили, что работаем в нормальных земных условиях. На Земле всегда есть хотя бы небольшое трение. В конце концов, когда вы в последний раз что-то толкали, и оно так и не остановилось?

На Земле трение действует как внешняя сила, заставляющая вещи переставать двигаться. Мяч никогда ни во что не врезается, и нет других сил, которые изменят направление движения. Учитывая все это, правильный ответ на эту задачу 3 , ящик остановится, но не изменит направление.

Вам может быть интересно, как Исаак Ньютон вычислил свой первый закон, учитывая то, как он жил на Земле. Отличный вопрос! Он размышлял о том, что на самом деле заставляло вещи переставать двигаться — кончалась ли у них энергия или на них воздействовало что-то вроде трения. Когда он решил, что это должна быть внешняя сила, он провел сравнительно бесконфликтный эксперимент, чтобы проверить свою гипотезу.

Пример 2. Шар на бесконечной плоскости без трения

Рассмотрим еще один пример первого закона Ньютона. Вместо того, чтобы пинать мяч на игровое поле, игрок пинает его на бесконечно длинную плоскость без трения. Если предположить, что мяч ни во что не врежется и никто его не коснется, что произойдет с мячом?

  1. Мяч остановится и изменит направление.
  2. Мяч не остановится, а изменит направление.
  3. Мяч остановится, но не изменит направление.
  4. Мяч не остановится и не изменит направление.

На мяч не действуют никакие внешние силы, поэтому, даже если он попытается остановиться, он не сможет. Он также не может ускорить себя. Мяч будет продолжать скользить по бесконечной плоскости с той же скоростью до самого конца Вселенной. У нас снова нет внешних сил, чтобы изменить направление движения мяча. Правильный ответ здесь 4 , коробка не остановится и не изменит направление.

Изучить Первый закон Ньютона об Альберте

Второй закон движения Ньютона
Второй закон движения Ньютона гласит, что результирующая сила, действующая на объект, равна произведению его массы на его ускорение.

Это, вероятно, тот, который наиболее часто и очевидно встречается в жизни. Вот почему более крупным автомобилям нужна дополнительная мощность, чтобы разогнаться до той же скорости. Это объясняет, почему толкать полный комод или книжный шкаф труднее, чем пустой. Это даже то, почему вы хотите, чтобы тяжелый конец был на вашей стороне веревки в перетягивании каната. Второй закон также является единственным с соответствующим уравнением.

Какова формула второго закона Ньютона?

Как мы уже говорили, второй закон Ньютона гласит, что результирующая сила, действующая на объект, равна произведению массы на ускорение. Естественно, уравнение показывает ту же информацию:

Второй закон Ньютона
F=ma

…где F — результирующая сила, m — масса, a — ускорение. Вы увидите, что это уравнение часто встречается на протяжении всей вашей карьеры физика в нескольких различных формах, поэтому лучше начать понимать его прямо сейчас.

Заинтересованы в лицензии школы Альберта?

Примеры законов Ньютона: второй закон движения Ньютона

Давайте рассмотрим несколько примеров второго закона Ньютона, чтобы продемонстрировать как концепцию, так и расчет.

Пример 1: Применение концепции

В какой из следующих ситуаций потребуется наибольшая результирующая сила?

  1. Малой массе сообщается небольшое ускорение
  2. Большой массе сообщается большое ускорение
  3. Большой массе придается малое ускорение
  4. Малой массе придается большое ускорение

Правильный ответ здесь 2 . Большой массе сообщается большое ускорение. Если мы вернемся ко второму закону Ньютона, то легко поймем, почему. Если чистая сила является произведением массы и ускорения, то увеличение любого из них приведет к увеличению общей необходимой силы. Следовательно, придание большой массе большого ускорения потребует наибольшей силы.

Скорее всего, если вы увидите подобные концептуальные вопросы в будущем, они будут использовать более конкретные примеры с использованием конкретных объектов и ускорений. В любом случае, смысл будет заключаться в том, чтобы выяснить комбинацию, которая создает наибольшее произведение массы и ускорения. Когда вы получаете такие вопросы, может быть лучше просто проверить цифры, особенно если вы смотрите на стандартизированный тест. 9{22}\text{ кг} проходит достаточно близко от звезды, чтобы почувствовать гравитационную силу в 100\text{ Н}, притягивающую его к звезде. Каково ускорение планеты при движении к звезде?

На данный момент, надеюсь, не слишком удивительно, что вы можете переставлять уравнения физики, чтобы решать их для разных значений. Мы уже много раз это видели, и мы собираемся сделать это здесь снова, чтобы использовать второй закон Ньютона для решения задачи ускорения.

Шаг 1. Определите, что вы знаете 92

Изучить Второй закон Ньютона об Альберте

Третий закон движения Ньютона
Третий закон Ньютона гласит, что на каждое действие существует равная и противоположная реакция.

Вы, наверное, слышали это в терминах таких вещей, как карма, но в физике это на самом деле относится к парам сил. Как правило, существуют уравновешенные силы (когда силы равны и противоположны и уравновешивают друг друга) и неуравновешенные силы (когда одна сила больше другой). Если вы внимательно присматривались, то могли догадаться, что Третий закон Ньютона касается в основном уравновешенных сил и уравновешенных пар сил. Причина, по которой силы всегда уравновешены, когда мы говорим о третьем законе Ньютона, заключается в том, что мы не рассматриваем силы, действующие на один объект, мы рассматриваем силу действия и соответствующую реакцию между двумя объектами.

Пары действие-противодействие

Пары действие-противодействие возникают, когда один объект прикладывает силу к другому. Третий закон Ньютона гласит, что если вы прикладываете силу к бейсбольному мячу, чтобы бросить его, то бейсбольный мяч действует на вас с такой же силой в противоположном направлении. Поначалу это может показаться немного странным. В конце концов, второй закон Ньютона только что сказал нам, что приложение силы создает ускорение.

Если вместо этого вы подумаете о том, что гравитация Земли еще не притянула вас к ее ядру, это может иметь немного больше смысла. Ваш вес создает силу, которая давит на землю, которая притянула бы вас к ядру Земли, если бы земля не оказывала на ваши ноги равную и противоположную силу. То же самое происходит, когда вы прислоняетесь к стене. Ваше тело прикладывает силу к стене, но стена способна отразить точно такую ​​же силу в ответ на третий закон Ньютона. В этом случае вы, прислонившись к стене, были бы действием, а стена, поддерживающая вас, была бы реакцией. Вместе они составляют пару действие-противодействие.

Заинтересованы в лицензии школы Альберта?

Примеры законов Ньютона: Третий закон движения Ньютона

Третий закон Ньютона задают только концептуально и обычно в связи с двумя другими законами. Давайте вместе рассмотрим несколько примеров.

Пример 1: Обнаружение собственной силы

В среднем Земля оказывает силу около 700\text{ Н} на каждого человека, прогуливающегося по ее поверхности. Какую силу вы оказываете на Землю?

Что ж, давайте подумаем. Третий закон Ньютона гласит, что каждое действие (Земля воздействует на вас) имеет равную и противоположную реакцию (вы воздействуете на Землю). По этой логике сила, которую вы оказываете на Землю, должна быть -700\text{ Н}. Вы можете увидеть, что на такие вопросы отвечает абсолютное значение этой силы (700\text{N} вместо -700\text{N}), но если вопрос конкретно не требует этого, лучше включить положительный или отрицательный знак по мере необходимости. 9{-13}\text{ Н}.

Этот побочный эффект третьего закона Ньютона, что вы так же сильны, как даже самые большие, самые крутые и самые загадочные тела во вселенной, и что они обладают силой только потому, что вы существуете, в некотором смысле является одним из самых крутых (хотя и наименее научные) уроки, которые мы можем извлечь из физики.

Изучите Третий закон Ньютона об Альберте

Приведение всех законов в действие

Хотите верьте, хотите нет, но вы наблюдали за взаимодействием всех трех этих законов и даже полагались на это взаимодействие всю свою жизнь. Пример, который мы будем использовать здесь, вряд ли попадется вам на глаза, но мы все равно будем использовать его, потому что в космосе очень легко игнорировать сопротивление воздуха и трение.

Примеры законов Ньютона с космическим исследователем

Давайте рассмотрим пример, чтобы показать, как все законы движения Ньютона работают вместе. Бесстрашный исследователь космоса покидает свой корабль, чтобы завершить необходимое техническое обслуживание. Они прикрепляются к кораблю страховочным шнуром, но не замечают, что он развязался там, где они привязывали его к кораблю. Когда они начинают использовать его, чтобы подтянуться к двери, они понимают, что он расстегнулся. Теперь они плывут вне досягаемости своего корабля и удаляются дальше. Как они могли спастись?

Закон инерции

Начнем с первого закона Ньютона. Если никакая внешняя сила не будет применена, они будут продолжать свой путь от своего корабля вечно… или пока не наткнутся на планету. В любом случае, это путешествие они вряд ли переживут. При этом ничто не заставляет их уходить быстрее, и у них остается много кислорода, чтобы все обдумать, так что это решаемая проблема. Поскольку у них нет ни воздуха, ни пола, по которым они могли бы толкаться, они не могут ни идти, ни плыть обратно на корабль. Им понадобится другой способ генерировать силу вне себя.

Пары действие-противодействие

Теперь давайте перейдем к третьему закону Ньютона. Этот говорит нам, что на каждое действие есть равное и противоположное противодействие. Хотя они ничего не могут сделать с собой, чтобы создать внешнюю силу, у них все еще есть вышедший из строя шнур безопасности… возможно, он все еще может их спасти. Если бы они бросили шнур так сильно, как только могли, в сторону от корабля, то та же сила применилась бы к нашему бесстрашному исследователю, подталкивающему их к кораблю.

Создание неуравновешенных сил

Следующий закон Ньютона. Бесстрашный исследователь прилагает все усилия, чтобы оторвать шнур от корабля. Относительно легкий шнур движется довольно быстро, в то время как они начинают возвращаться несколько медленнее, так как большая масса с той же силой всегда будет получать меньшее ускорение. Тем не менее, они возвращаются в безопасное место на своем корабле.

Объект в движении остается в движении

Наконец, снова первый закон Ньютона. Между нашим космическим авантюристом и его кораблем нет ничего стоящего (или плавающего). Вокруг нет ничего, что могло бы применить внешнюю силу ни к авантюристу, ни к его кораблю. Все, что осталось, — осторожно столкнуться с кораблем, а затем проползти по его внешней стороне к двери и в безопасное место.

На протяжении всей своей карьеры физика вы наверняка сталкивались с подобными законами Ньютона. Вы можете увидеть людей, парящих в космосе, застрявших в озере без трения или катающихся на скейтбордах без трения. Если у вас есть один человек с предметом или пара людей, застрявших в такой опасной ситуации, Законы Ньютона могут вмешаться, чтобы спасти положение.

Изучите основанную на феноменах практику Альберта

Почему важны законы Ньютона?

Вы изучили законы движения Ньютона и изучили различные примеры законов Ньютона. От помощи в определении силы, необходимой для создания определенного движения, до спасения жизней бесстрашных астронавтов — эти законы будут возникать в вашей повседневной жизни снова и снова. Они также составляют основу почти всего, что вы изучаете здесь и до конца своей физической карьеры. Исаак Ньютон произвел революцию в нашем представлении о том, как движутся вещи — большие и малые. За последнее столетие потребовалось лишь небольшое обновление, чтобы учесть очень-очень маленькое и очень-очень большое. То, как мы воспринимаем мир, с которым взаимодействуем, было сформировано законами Ньютона, и, надеюсь, вы сможете начать замечать их и использовать с пользой, что сделает вашу жизнь немного проще.

Законы интимной любви Ньютона. 5 июля 1687 года сэр Исаак Ньютон… | by Bharat

5 июля 1687 года сэр Исаак Ньютон опубликовал свою книгу: Philosophiæ Naturalis Principia Mathematica, в которой он представил новые идеи в физике, математике, наблюдениях гравитации и вывел печально известные три закона движения.

В качестве простого упражнения на умственную бесполезность и чувство юмора я попытался объяснить эту безумную мелочь под названием Любовь, используя законы движения Ньютона для описания интимных отношений между двумя влюбленными людьми.

Первый закон

Первый закон движения Ньютона: Каждое тело остается в состоянии покоя или равномерного движения до тех пор, пока на него не действует результирующая сила .

Этот Первый Закон можно переписать следующим образом:

Каждый человек продолжает находиться в Стабильном Состоянии изоляции или любовных отношений, если на него не воздействует Внешнее Жизненное Событие .

Под таблицей S Состояние я имею в виду, что они либо уже могут быть «счастливыми и довольными», либо в равной степени они могут быть «грустными и несчастными», независимо от того, одиноки ли они или уже состоят в отношениях.

Внешнее жизненное событие означает в контексте любого состояния, в котором они находятся в настоящее время. Например:

  • В одиночестве: жизнь знакомит их с новым человеком, в которого они влюбляются.
  • Существующие отношения: Их отношения распадаются, потому что Жизнь представляет третьего человека, который соблазняет одного из них, или Жизнь создает личное бедствие для одного или обоих из них такой жестокой силы, что один из них решает прекратить отношения.

Второй закон

Второй закон Ньютона: Ускорение (a) объекта напрямую зависит от чистой силы (F), действующей на объект, и обратно пропорционально массе (m) объекта.

Этот закон обычно представляется уравнением: F=m * a

Но для наших целей я предпочитаю следующее (и более точное) представление: a = F / m

Второй закон может переписать следующим образом:

Близость двух людей напрямую зависит от Чистой Эмоциональной Любви между ними и, наоборот, от объединенной Эмоциональной Массы их обоих. По мере того, как Эмоциональная Любовь увеличивается, Близость между ними увеличивается. По мере того, как Эмоциональная Масса одного из них увеличивается, Близость между ними уменьшается!

Чтобы завершить аналогию, уравнение, представляющее Второй Закон Эмоциональной Любви:

Близость = Чистая Эмоциональная Любовь / Эмоциональная Масса

Поясним приведенные выше термины.

Чистая эмоциональная любовь : Под эмоциональной любовью я подразумеваю взаимное влечение между двумя людьми, скажем: Алексом и Брендой. В этом случае чистая эмоциональная любовь означает притяжение между двумя людьми без учета кого-либо еще. Итак, если появится Чарльз и соблазнит Бренду своими глубокими привлекательными качествами, то чистое влечение между Алексом и Брендой может легко уменьшиться.

Комбинированная эмоциональная масса : Я полагаю, что у людей есть нечто, называемое Эмоциональной массой, которое показывает отсутствие у них восприимчивости или эмоциональной инерции к внешним изменениям, например: новое любящее намерение, обращение с изменяющим жизнь событием или что-то другое. У некоторых людей есть сила характера и любовь к себе, чтобы противостоять тому, что бросает им жизнь, поэтому они продолжают ставить своих любящих партнеров на первое место, не ставя под угрозу себя.

Близость : Как и ускорение, близость всегда направлена ​​между двумя людьми. Это то, что глубже любви. Другими словами, Любовь состоит из Близости, которую разделяют два человека. Односторонняя близость — это безответная любовь. Близость — это близость между двумя людьми, при которой они испытывают высокую степень сочувствия друг к другу и кажутся эмоционально/духовно связанными независимо от физического расстояния между ними.

Чтобы расширить наш надуманный пример: если бы Бренда действительно любила себя, то она смогла бы игнорировать ухаживания Чарльза и поддерживать любовные отношения, которые у нее сейчас есть с Алексом. Другими словами, если у Бренды высокая самооценка, то у нее высокий уровень Эмоциональной Массы, поэтому любая новая Эмоциональная Любовь от Чарльза приведет только к низкому уровню Близости между ней и Чарльзом. Ее крепкие интимные отношения с Алексом не ослабевают.

Кроме того, второй закон Ньютона часто записывают для ускорения под действием земного притяжения g следующим образом: F = mg

В этой форме Близость подобна Гравитации. Это константа, хотя люди и знают о ней, но это одна из тех редких вещей, которые можно и нужно принимать как должное. Во время стихийного бедствия, такого как извержение вулкана, в воздух выбрасываются миллионы тонн мусора. Тем не менее, из-за невероятной массы Земли гравитация вскоре притягивает эти летучие высокоскоростные частицы, и со временем формируется новое равновесие и восстанавливается мир.

Близость – это то же самое, что и Гравитация. В очень нестабильные времена Жизнь буквально разорвет двух людей на части. Но если они полагаются на свою Близость, основанную на их Объединенной Эмоциональной Массе, тогда они могут и переждать бурю, и преодолеть все, что подбрасывает им Жизнь.

Третий закон

Третий закон движения Ньютона: На каждое действие есть равное и противоположное противодействие.

Третий закон можно переписать следующим образом:

На каждое интимное действие существует равная и противоположная интимная реакция.

В ньютоновских терминах, когда ноутбук лежит на столе, он не двигается. Он идеально сбалансирован. Стабильный. Это связано с тем, что направленная вниз сила земного притяжения идеально уравновешивается толканием стола вверх. Если бы это было не так, ноутбук сломал бы стол или, что еще хуже, стол подбросил бы ноутбук вверх на орбиту Земли!

Я предполагаю, что когда два человека влюблены, стабильность их отношений основана на том, что каждый из них постоянно действует и реагирует на потребности друг друга, так что их совместная цель и намерение – поддерживать отношения. Они оба путешествуют по одному и тому же пути (или орбите). И ни один не подталкивает и не тянет другого в одностороннем направлении. Это не означает, что между ними нет различий. Наоборот, этот закон активно зависит как от поиска различий друг в друге, так и от совместной работы по сглаживанию этих различий, чтобы сохранялась стабильность и направление их путешествия по Жизни. Научитесь отмечать эти различия и несовершенства, идя на компромисс и желая помогать друг другу расти.

Выводы

Мы живем в одиночестве или вместе с другими. Единственная константа заключается в том, что мы знаем, что Вселенная (или Бог наверху) будет ставить препятствия и возможности на нашем пути. Я не могу объяснить, почему происходят эти события, но, возможно, с помощью замечательной работы Ньютона мы сможем лучше справиться с этими судьбоносными событиями — здесь и сейчас.

Мир.

Классическая физика

Прежде чем перечислять законы Ньютона, стоит пояснить, что понимается под словом «закон». В науке сегодня слова закон , теория и гипотеза примерно синонимичны. Теория — это описательный или объяснительный отчет о чем-либо. Слово гипотеза обычно относится к теории, которая еще не была тщательно проверена и подтверждена наблюдениями или экспериментами. Слово закон обычно относится к хорошо подтвержденной теории, описывающей некоторую закономерность в природе. Однако эти термины не всегда используются точным или последовательным образом (например, «теория струн» — полностью непроверенная гипотеза), поэтому не слишком полагайтесь на эти различия. А пока просто имейте в виду, что слова теория , гипотеза и закон по существу являются синонимами: все они относятся к описательным или объяснительным объяснениям (хотя они означают разную степень поддержки наблюдениями).

С другой стороны, когда Ньютон использовал слово «закон» для описания математических закономерностей, открытых им в природе, он не использовал это слово просто как синоним «теории». Подобно своим предшественникам Кеплеру и Галилею, Ньютон был набожным христианином и рассматривал естественные закономерности как результат установленных Богом принципов, управляющих творением. В конце своего великого опуса, The Mathematical Principles of Natural Philosophy (1846), Ньютон явно приписывает Богу порядок, который можно найти в природе:

Эта прекраснейшая система солнца, планет и комет могла возникнуть только по совету и владычеству разумного и могущественного Существа. И если неподвижные звезды являются центрами других подобных систем, то они, образованные таким же мудрым советом, должны все подчиняться владычеству Единого… Это Существо управляет всеми вещами не как душа мира, но как Господь. над всем… И из его истинного владычества следует, что истинный Бог есть живое, разумное и могущественное Существо; и, исходя из других его совершенств, что он высший или самый совершенный. Он вечен и бесконечен, всемогущ и всеведущ; то есть его продолжительность простирается от вечности до вечности; его присутствие от бесконечности до бесконечности; он управляет всем и знает все, что можно или можно сделать. Весь текст доступен здесь.

Законы движения Ньютона касаются воздействия сил на физические объекты. Сила — это просто толчок или тяга. Силы являются векторными величинами: они имеют как величину, так и направление. Чистая сила (полная сила), действующая на объект, представляет собой сумму всех сил, действующих на него. (Объяснение того, как можно сложить векторы, см. на предыдущей странице.)

Первый закон движения Ньютона гласит, что скорость объекта (скорость и направление движения) не изменяется до тех пор, пока результирующая сила, действующая на объект равен нулю. Первый закон Ньютона также известен как 9-й закон.0124 закон инерции .

Первый закон Ньютона логически вытекает из второго закона, поскольку последний подразумевает, что ускорение равно нулю, когда результирующая сила равна нулю. Таким образом, первый закон можно рассматривать как частный случай второго закона: случай, когда F = 0,

. Второй закон Ньютона гласит, что результирующая сила, действующая на объект, равна произведению массы объекта на его ускорение. : F = ма В приведенном выше уравнении F представляет собой величину (силу) чистой силы, м — масса объекта, а а — величина (скорость) его ускорения.

Стандартная единица силы, называемая ньютон (обозначается заглавной буквой «Н»), представляет собой количество силы, необходимой для ускорения одного килограмма массы со скоростью один метр в секунду в секунду (т. е. его скорость увеличивается на 1 м/с каждую секунду): 1 Н = 1 кг м/с 2

Предположим, что на тело массой 3 кг, которое изначально находилось в состоянии покоя, действует результирующая сила 6 Н. С какой скоростью будет двигаться объект через 10 секунд? Чтобы ответить на этот вопрос, мы сначала должны определить ускорение, используя второй закон движения Ньютона:

сила = масса × ускорение
6 Н = 3 кг ×
6 Н/3 кг =
2 м/с 2 = а

Итак, объект будет ускоряться со скоростью 2 м/с 2 , что означает, что каждую секунду он будет двигаться на 2 м/с быстрее. Поскольку первоначально он находился в состоянии покоя и ускорялся с такой скоростью в течение 10 секунд, его конечная скорость будет равна 20 м/с в любом направлении, в котором его толкает сила.

Третий закон движения Ньютона гласит, что всякий раз, когда один объект воздействует на другой объект, второй объект одновременно воздействует на первый объект. Эти две силы равны по величине, но противоположны по направлению.

Например, земля притягивает падающее яблоко с направленной вниз гравитационной силой; яблоко одновременно тянет землю вверх с силой равной величины. (Конечно, это восходящее притяжение не сдвинет землю заметно с места, потому что земля имеет такую ​​большую массу.)

Хотя силы равны и противоположны, они не компенсируют друг друга, так как они действуют на разные объекты. .

Из трех законов Ньютона вместе следует закон сохранения импульса , в котором говорится, что общий импульс физической системы (любого набора физических объектов) сохраняется (не изменяется), пока на систему не действуют внешние силы. Другими словами, общий импульс системы (сумма векторов импульса для каждого объекта в системе) всегда остается одним и тем же, если объект вне системы не оказывает силы на один или несколько объектов внутри системы. Это верно для полного линейного количества движения и для полного углового момента системы.

Вот простой пример, иллюстрирующий, как закон сохранения импульса следует из законов Ньютона. Предположим, что камень массой 2 кг движется вправо со скоростью 5 м/с, а камень массой 3 кг движется влево со скоростью 4 м/с. Импульс меньшего камня (его масса × скорость) составляет 2 кг × 5 м/с = 10 кг м/с вправо. Импульс большего камня составляет 3 кг × 4 м/с = 12 кг м/с влево. Поскольку два вектора импульса указывают в противоположных направлениях, сложение этих векторов вместе дает вектор длины 2, указывающий влево. (Объяснение сложения векторов см. на предыдущей странице.) Итак, общий импульс системы двух пород, первоначально, составляет 2 кг м/с влево.

Теперь предположим, что камни воздействуют друг на друга. По третьему закону Ньютона эти силы должны быть равны по величине, но противоположны по направлению. Например, если большой камень действует с силой 30 ньютонов, толкая маленький камень влево, маленький камень должен прилагать 30 ньютонов, толкая большой камень вправо. Давайте посмотрим, что произойдет, если камни воздействуют друг на друга с такой силой в течение 1 секунды. Сначала рассмотрим, что происходит с маленьким камнем массой 2 кг. По второму закону Ньютона:

сила = масса × ускорение
30 Н = 2 кг × ускорение
15 м/с 2 = ускорение

Маленький камень будет ускоряться со скоростью 15 м/с 2 влево — направление, в котором его толкают. Другими словами, его скорость влево будет увеличиваться на 15 м/с каждую секунду. Поскольку изначально он двигался к вправо со скоростью 5 м/с, через 1 секунду он будет двигаться со скоростью 10 м/с влево. Следовательно, его конечный импульс будет 2 кг × 10 м/с = 20 кг м/с влево.

Далее давайте посмотрим, что происходит с большим камнем массой 3 кг:

сила = масса × ускорение
30 Н = 3 кг × ускорение
10 м/с 2 = ускорение

Большой камень будет двигаться со скоростью 10 м/с 2 вправо . Поскольку изначально он двигался к влево со скоростью 4 м/с, через 1 секунду он будет двигаться вправо со скоростью 6 м/с. Следовательно, его конечный импульс будет равен 3 кг × 6 м/с = 18 кг м/с вправо.

Итак, конечный импульс маленького камня равен 20 кг м/с влево, а конечный импульс большого камня равен 18 кг м/с вправо. Каков общий импульс системы из двух камней теперь, после того как камни оказали воздействие друг на друга? Сложение этих конечных векторов импульса вместе дает вектор длины 2, указывающий влево.

Оставить комментарий